Está en la página 1de 312

ENAM 2024:

HEMATOLOGÍA
M.C. LUIS FAJARDO CORONATTA
ESPECIALISTA DEN MEDICINA INTERNA
• Varón de 48 años hace 4 días diagnóstico de piocolecisto, actualmente
en sepsis. Presenta hematuria y lesiones purpúricas en miembros
inferiores. Hto 28%, leucocitos: 23,300/mm3; plaquetas: 75,000/mm3;
protrombina, TTPa y tiempo de trombina prolongados fibrinógeno
disminuido. ¿Cuál es el probable diagnóstico?
a. Déficit de vitamina K
b. Hemofilia A
c. Púrpura trombocitopénica autoinmune
d. Coagulación intravascular diseminada
e. Mieloma múltiple
• Varón de 48 años hace 4 días diagnóstico de piocolecisto, actualmente
en sepsis. Presenta hematuria y lesiones purpúricas en miembros
inferiores. Hto 28%, leucocitos: 23,300/mm3; plaquetas: 75,000/mm3;
protrombina, TTPa y tiempo de trombina prolongados fibrinógeno
disminuido. ¿Cuál es el probable diagnóstico?
a. Déficit de vitamina K
b. Hemofilia A
c. Púrpura trombocitopénica autoinmune
d. Coagulación intravascular diseminada
e. Mieloma múltiple
Evaluación y manejo de la coagulación intravascular diseminada (CID) en
adultos (Uptodate)
• En el resultado de un hemograma se encuentra: Hemoglobina
disminuida, microcitosis, VCM disminuido y ferritina sérica disminuida.
¿A qué tipo de anemia corresponde?
a. Ferropénica
b. Megaloblástica
c. Hemolítica
d. Falciforme
e. Inflamatoria
• En el resultado de un hemograma se encuentra: Hemoglobina
disminuida, microcitosis, VCM disminuido y ferritina sérica disminuida.
¿A qué tipo de anemia corresponde?
a. Ferropénica
b. Megaloblástica
c. Hemolítica
d. Falciforme
e. Inflamatoria
Microcitosis/Anemia microcítica (Uptodate)
• Varón de 45 años, alcohólico crónico, hace 6 meses debilidad y
parestesias de extremidades, disnea al esfuerzo. Al examen: pálido,
pérdida de la sensibilidad termoalgésica, cuadriparesia espástica leve a
predominio distal, Hb: 7 gr/dL, VCM: 120 fl. ¿Cuál sería el diagnóstico
más probable?
a. Síndrome de Guillain Barre
b. Encefalopatía hepática
c. Déficit de vitamina B12
d. Anemia sideroblástica
e. Hipotiroidismo
• Varón de 45 años, alcohólico crónico, hace 6 meses debilidad y
parestesias de extremidades, disnea al esfuerzo. Al examen: pálido,
pérdida de la sensibilidad termoalgésica, cuadriparesia espástica leve a
predominio distal, Hb: 7 gr/dL, VCM: 120 fl. ¿Cuál sería el diagnóstico
más probable?
a. Síndrome de Guillain Barre
b. Encefalopatía hepática
c. Déficit de vitamina B12
d. Anemia sideroblástica
e. Hipotiroidismo
Manifestaciones clínicas y diagnóstico de deficiencia de vitamina B12 y
folato (Uptodate)
• Mujer de 40 años presenta anorexia, pérdida de peso y diarrea crónica.
Antecedente: hace 4 años fiebre tifoidea complicada, que obligó a
resección del íleon terminal. Al examen: palidez marcada de piel,
glositis, queilosis, ictericia. ¿Cuál es el diagnóstico más probable?
a. Anemia megaloblástica
b. Talasemia
c. Anemia ferropénica
d. Esferocitosis
e. Anemia sideroblástica
• Mujer de 40 años presenta anorexia, pérdida de peso y diarrea crónica.
Antecedente: hace 4 años fiebre tifoidea complicada, que obligó a
resección del íleon terminal. Al examen: palidez marcada de piel,
glositis, queilosis, ictericia. ¿Cuál es el diagnóstico más probable?
a. Anemia megaloblástica
b. Talasemia
c. Anemia ferropénica
d. Esferocitosis
e. Anemia sideroblástica
Causas y fisiopatología de las deficiencias de vitamina B12 y folato (Uptodate)
• Niño de 8 años traído a consulta por dolor abdominal crónico e
hiporexia, madre manifiesta que es hiperactivo en la escuela. Al
examen: palidez de piel y mucosas, talla baja para la edad. Hb de 10
g/dL, microcitosis e hipocromía. Vive en Cerro de Pasco desde los 6
meses. ¿Cuál es el diagnóstico probable?
a. Intoxicación por plomo
b. Esferocitosis hereditaria
c. Deficiencia de hierro
d. Deficiencia de zinc
e. Deficiencia de cobalamina
• Niño de 8 años traído a consulta por dolor abdominal crónico e
hiporexia, madre manifiesta que es hiperactivo en la escuela. Al
examen: palidez de piel y mucosas, talla baja para la edad. Hb de 10
g/dL, microcitosis e hipocromía. Vive en Cerro de Pasco desde los 6
meses. ¿Cuál es el diagnóstico probable?
a. Intoxicación por plomo
b. Esferocitosis hereditaria
c. Deficiencia de hierro
d. Deficiencia de zinc
e. Deficiencia de cobalamina
Intoxicación por plomo infantil: manifestaciones clínicas y diagnóstico (Uptodate)
• Varón de 56 años, diabético acude a control. Antecedente filtrado
glomerular 20 ml/min/1.73 m2, anemia en tratamiento con eritropoyetina.
Laboratorio: hemoglobina 9.7g/dl, hematocrito 32.4%, ferritina 56 ng/ml,
índice de saturación de la transferrina del 12%. ¿Qué tratamiento se
debe continuar?
a. Administrar hierro y mantener la misma dosis de eritropoyetina.
b. Aumentar solo la dosis de eritropoyetina.
c. Suspender la eritropoyetina y administrar hierro.
d. Transfundir paquete globular
e. Administrar corticoides
• Varón de 56 años, diabético acude a control. Antecedente filtrado
glomerular 20 ml/min/1.73 m2, anemia en tratamiento con eritropoyetina.
Laboratorio: hemoglobina 9.7g/dl, hematocrito 32.4%, ferritina 56 ng/ml,
índice de saturación de la transferrina del 12%. ¿Qué tratamiento se
debe continuar?
a. Administrar hierro y mantener la misma dosis de eritropoyetina.
b. Aumentar solo la dosis de eritropoyetina.
c. Suspender la eritropoyetina y administrar hierro.
d. Transfundir paquete globular
e. Administrar corticoides
Tratamiento de la anemia en la enfermedad renal crónica sin diálisis (Uptodate)
• Gestante de 6 meses, consulta por palidez. Estudio hemático:
hemoglobina: 8 g/dl. Constantes corpusculares disminuidas. Anisocitosis
++, hipocromía ++, presencia de microcitosis y reticulocitos disminuidos.
Estos hallazgos son compatibles con una anemia:
a. Ferropénica
b. Aplásica
c. Perniciosa
d. Hemolítica
e. Sideroblástica
• Gestante de 6 meses, consulta por palidez. Estudio hemático:
hemoglobina: 8 g/dl. Constantes corpusculares disminuidas. Anisocitosis
++, hipocromía ++, presencia de microcitosis y reticulocitos disminuidos.
Estos hallazgos son compatibles con una anemia:
a. Ferropénica
b. Aplásica
c. Perniciosa
d. Hemolítica
e. Sideroblástica
Anemia en el embarazo (Uptodate)
• Varón de 40 años, alcohólico, obeso. Se somete a estudios encontrando
los siguientes hallazgos relevantes: palidez, hemoglobina: 9 g/dl,
macrocitosis. vitamina B12 disminuida, ácido fólico normal. Biopsia
gástrica: gastritis crónica con atrofia de las glándulas fúndicas. ¿Cuál de
las siguientes sustancias estaría deficiente y explicaría el cuadro?
a. Factor intrínseco
b. Gastrina
c. Urogastrona
d. Defensina
e. Hepcidina
• Varón de 40 años, alcohólico, obeso. Se somete a estudios encontrando
los siguientes hallazgos relevantes: palidez, hemoglobina: 9 g/dl,
macrocitosis. vitamina B12 disminuida, ácido fólico normal. Biopsia
gástrica: gastritis crónica con atrofia de las glándulas fúndicas. ¿Cuál de
las siguientes sustancias estaría deficiente y explicaría el cuadro?
a. Factor intrínseco
b. Gastrina
c. Urogastrona
d. Defensina
e. Hepcidina
Causas y fisiopatología de las deficiencias de vitamina B12 y folato (Uptodate)
• Mujer de 44 años, refiere hace 2 semanas equimosis múltiples, debilidad
y tos intensa con esputo amarillento. Al examen: T° 38,4 °C y crepitantes
en ambos campos pulmonares. Laboratorio: leucocitos 1.400/mm3; Hb
7,2 g/dl; plaquetas:35.000/mm3. En su anamnesis ¿Cuál es la
información que usted debe buscar para llegar al diagnóstico?
a. Exposición previa a medicamentos
b. Antecedentes familiares de anemias
c. Hábitos dietéticos permanentes
d. Infecciones bacterianas recientes
e. Historia de sangrado reciente
• Mujer de 44 años, refiere hace 2 semanas equimosis múltiples, debilidad
y tos intensa con esputo amarillento. Al examen: T° 38,4 °C y crepitantes
en ambos campos pulmonares. Laboratorio: leucocitos 1.400/mm3; Hb
7,2 g/dl; plaquetas:35.000/mm3. En su anamnesis ¿Cuál es la
información que usted debe buscar para llegar al diagnóstico?
a. Exposición previa a medicamentos
b. Antecedentes familiares de anemias
c. Hábitos dietéticos permanentes
d. Infecciones bacterianas recientes
e. Historia de sangrado reciente
Abordaje del adulto con pancitopenia (Uptodate)
Abordaje del adulto con pancitopenia (Uptodate)
Abordaje del adulto con pancitopenia (Uptodate)
Abordaje del adulto con pancitopenia (Uptodate)
Abordaje del adulto con pancitopenia (Uptodate)
• Mujer de 25 años refiere desde hace 3 años artralgias. Examen: palidez,
no deformidades articulares. Laboratorio: leucocitos 5.890/mm3; Hb 12,5
g/dl; plaquetas 217.000/mm3. Frotis de sangre periférica: hipocromía y
microcitosis. Hierro total y ferritina normales. Electroforesis de
hemoglobina: HbA1: 93%, HbA2: 5,8% y HbF:1,2%. ¿Cuál es el
diagnóstico más probable?
a. β-talasemia menor
b. Anemia hemolítica autoinmune
c. Anemia ferropénica
d. Infección por Plasmodium vivax
e. Infección por Plasmodium falciparum
• Mujer de 25 años refiere desde hace 3 años artralgias. Examen: palidez,
no deformidades articulares. Laboratorio: leucocitos 5.890/mm3; Hb 12,5
g/dl; plaquetas 217.000/mm3. Frotis de sangre periférica: hipocromía y
microcitosis. Hierro total y ferritina normales. Electroforesis de
hemoglobina: HbA1: 93%, HbA2: 5,8% y HbF:1,2%. ¿Cuál es el
diagnóstico más probable?
a. β-talasemia menor
b. Anemia hemolítica autoinmune
c. Anemia ferropénica
d. Infección por Plasmodium vivax
e. Infección por Plasmodium falciparum
Diagnóstico de talasemia (adultos y niños) (Uptodate)
• Mujer de 60 años desde hace 4 meses presenta astenia, dolor de
espalda y extremidades al deambular, Al examen: PA 120/80 mm Hg,
FC: 78 x', FR: 20 x', T:38.5°C. Palidez de piel, hepatoesplenomegalia,
Laboratorio: Leucocitosis, anemia. Frotis: Leucocitos: bastones de Auer.
¿Cuál es el diagnóstico más probable?
a. Leucemia mieloide aguda
b. Leucemia linfática crónica
c. Linfoma
d. Mieloma múltiple
e. Leucemia de células plasmáticas
• Mujer de 60 años desde hace 4 meses presenta astenia, dolor de
espalda y extremidades al deambular, Al examen: PA 120/80 mm Hg,
FC: 78 x', FR: 20 x', T:38.5°C. Palidez de piel, hepatoesplenomegalia,
Laboratorio: Leucocitosis, anemia. Frotis: Leucocitos: bastones de Auer.
¿Cuál es el diagnóstico más probable?
a. Leucemia mieloide aguda
b. Leucemia linfática crónica
c. Linfoma
d. Mieloma múltiple
e. Leucemia de células plasmáticas
Leucemia mieloide aguda: manifestaciones clínicas, características patológicas y
diagnóstico (Uptodate)
• Mujer de 60 años gastrectomizada, presenta debilidad, palpitaciones,
disnea, siendo diagnosticada de anemia ¿Qué tipo de anemia es?
a. Microcítica ,hipocromica
b. Normocítica, normocrómica
c. Macrocítica, hipercrómica
d. Microcítica, normocrómica
e. Macrocítica, hipocrómica
• Mujer de 60 años gastrectomizada, presenta debilidad, palpitaciones,
disnea, siendo diagnosticada de anemia ¿Qué tipo de anemia es?
a. Microcítica ,hipocromica
b. Normocítica, normocrómica
c. Macrocítica, hipercrómica
d. Microcítica, normocrómica
e. Macrocítica, hipocrómica
Evaluación nutricional en pacientes con gastrectomía total y parcial por
adenocarcinoma gástrico

http://www.scielo.org.pe/scielo.php?script=sci_arttext&pid=S1022-51292008000300005&lng=es
• Varón de 56 años, con lumbalgia crónica. En estudios de laboratorio:
plasmocitosis medular deI 20%, Ig sérica: 2,5 g/L. Radiografía: lesiones
óseas líticas en columna vertebral. El diagnóstico es:
a. Plasmocitosis reactiva
b. Mieloma múltiple
c. Enfermedad ósea de Payet
d. Amiloidosis primaria
e. Gammapatía monoclonal de significado incierto
• Varón de 56 años, con lumbalgia crónica. En estudios de laboratorio:
plasmocitosis medular deI 20%, Ig sérica: 2,5 g/L. Radiografía: lesiones
óseas líticas en columna vertebral. El diagnóstico es:
a. Plasmocitosis reactiva
b. Mieloma múltiple
c. Enfermedad ósea de Payet
d. Amiloidosis primaria
e. Gammapatía monoclonal de significado incierto
Mieloma múltiple: características clínicas, manifestaciones de laboratorio y
diagnóstico (Uptodate)
• Paciente varón de 20 años, de raza negra, al ir de viaje a la sierra se
pone pálido y tiene disnea. ¿Cuál es el diagnóstico más probable?
• Anemia falciforme
• Anemia megaloblástica
• Leucemia promielocítica aguda
• Anemia hemolítica autoinmune
• Anemia refractaria
• Paciente varón de 20 años, de raza negra, al ir de viaje a la sierra se
pone pálido y tiene disnea. ¿Cuál es el diagnóstico más probable?
• Anemia falciforme
• Anemia megaloblástica
• Leucemia promielocítica aguda
• Anemia hemolítica autoinmune
• Anemia refractaria
http://www.scielo.cl/scielo.php?script=sci_arttext&pid=S0370-
41062018000400525&lng=es. http://dx.doi.org/10.4067/S0370-41062018005000604.
• Paciente con válvula protésica anticoagulado con warfarina, acude por
sangrado gingival. Exámenes auxiliares: INR: 8. ¿Cuál es la medida a
seguir?
a. Suspender warfarina
b. Suspender warfarina y administrar vitamina K
c. Suspender warfarina y administrar protamina
d. Suspender warfarina y administrar plasma fresco congelado
e. Suspender warfarina y administrar crioprecipitado
• Paciente con válvula protésica anticoagulado con warfarina, acude por
sangrado gingival. Exámenes auxiliares: INR: 8. ¿Cuál es la medida a
seguir?
a. Suspender warfarina
b. Suspender warfarina y administrar vitamina K
c. Suspender warfarina y administrar protamina
d. Suspender warfarina y administrar plasma fresco congelado
e. Suspender warfarina y administrar crioprecipitado
Manejo del sangrado asociado a warfarina o INR supraterapéutico (uptodate)
• Varón de 50 años presenta astenia progresiva, falta de concentración y
de memoria, insomnio, palpitaciones, acúfenos, cefalea, parestesias. Al
examen: PA:120/60 mmHg Afebril, piel pálida amarillenta, lengua
depapilada. Neurológico: paraparesia, marcha atáxica. Laboratorio:
Hb:6g/dl, Hto: 18%, Leucocitos: 2500/μl, Plaquetas: 60,000/μl, VCM: 102
fl. Lámina periférica, macrocitos ovalados, anisocitosis, poiquilocitosis.
¿Cuál es el tipo de anemia?
a. Sideroblástica
b. Megaloblástica
c. Ferropénica
d. Falciforme
e. Talasémica
• Varón de 50 años presenta astenia progresiva, falta de concentración y
de memoria, insomnio, palpitaciones, acúfenos, cefalea, parestesias. Al
examen: PA:120/60 mmHg Afebril, piel pálida amarillenta, lengua
depapilada. Neurológico: paraparesia, marcha atáxica. Laboratorio:
Hb:6g/dl, Hto: 18%, Leucocitos: 2500/μl, Plaquetas: 60,000/μl, VCM: 102
fl. Lámina periférica, macrocitos ovalados, anisocitosis, poiquilocitosis.
¿Cuál es el tipo de anemia?
a. Sideroblástica
b. Megaloblástica
c. Ferropénica
d. Falciforme
e. Talasémica
Manifestaciones clínicas y diagnóstico de deficiencia de vitamina B12 y
folato (uptodate)
• Según el sistema de estadificación de Ann Arbor para un paciente con
linfadenopatía generalizada y alta sospecha de linfoma No Hodgkin.
¿Cuál es el síntoma B?
a. Diaforesis profusa nocturna en el mes anterior
b. Dolor en hemiabdomen superior
c. Diarrea crónica
d. Infecciones recurrentes en el mes previo
e. Púrpura
• Según el sistema de estadificación de Ann Arbor para un paciente con
linfadenopatía generalizada y alta sospecha de linfoma No Hodgkin.
¿Cuál es el síntoma B?
a. Diaforesis profusa nocturna en el mes anterior
b. Dolor en hemiabdomen superior
c. Diarrea crónica
d. Infecciones recurrentes en el mes previo
e. Púrpura
Presentación clínica y evaluación inicial del linfoma no Hodgkin (uptodate)
• Varón de 76 años, diabético de larga data que consulta por
palpitaciones. Al examen se encuentra arritmia cardiaca y pulso
deficitario se inicia tratamiento con warfarina. ¿Cuál es el nivel
terapéutico por alcanzar en el INR para una adecuada anticoagulación?
a. 2a3
b. 3a4
c. 1a2
d. 4a5
e. 0a1
• Varón de 76 años, diabético de larga data que consulta por
palpitaciones. Al examen se encuentra arritmia cardiaca y pulso
deficitario se inicia tratamiento con warfarina. ¿Cuál es el nivel
terapéutico por alcanzar en el INR para una adecuada anticoagulación?
a. 2a3
b. 3a4
c. 1a2
d. 4a5
e. 0a1
Fibrilación auricular en adultos: uso de anticoagulantes orales (uptodate)
• Mujer de 22 años consulta por astenia progresiva, palpitaciones y
cefalea. Examen: palidez e ictericia. Laboratorio: Hb 9gr/dl, aumento del
recuento de reticulocitos (IR >5%), incremento del LDH, disminución de
haptoglobina ¿Cuál es el diagnóstico más probable?
a. Mieloptisis
b. Anemia hemolítica
c. Síndrome mielodisplásico
d. Anemia ferropénica
e. MIelofibrosis
• Mujer de 22 años consulta por astenia progresiva, palpitaciones y
cefalea. Examen: palidez e ictericia. Laboratorio: Hb 9gr/dl, aumento del
recuento de reticulocitos (IR >5%), incremento del LDH, disminución de
haptoglobina ¿Cuál es el diagnóstico más probable?
a. Mieloptisis
b. Anemia hemolítica
c. Síndrome mielodisplásico
d. Anemia ferropénica
e. MIelofibrosis
Diagnóstico de anemia hemolítica en adultos (uptodate)
• El cromosoma Philadelfia se presenta en la leucemia:
a. Mielocítica aguda
b. Mielomonocítica
c. A células plasmáticas
d. Megaloblástica
e. Mieloide crónica
• El cromosoma Philadelfia se presenta en la leucemia:
a. Mielocítica aguda
b. Mielomonocítica
c. A células plasmáticas
d. Megaloblástica
e. Mieloide crónica
Manifestaciones clínicas y diagnóstico de la leucemia mieloide
crónica (uptodate)
• Varón de 67 años refiere que el “útlimo mes le duelen todos los huesos”.
Hb: 9.1gr/dl. VSG: 90mmHr, proteinograma electroforético con pico
monoclonal, calcio 12.5mg/dl. ¿Cuál es el diagnóstico más probable?
a. Mieloma múltiple
b. Insuficiencia renal crónica
c. Cáncer de colon
d. Carcinoma de próstata
e. Lupus eritematoso sistémico
• Varón de 67 años refiere que el “útlimo mes le duelen todos los huesos”.
Hb: 9.1gr/dl. VSG: 90mmHr, proteinograma electroforético con pico
monoclonal, calcio 12.5mg/dl. ¿Cuál es el diagnóstico más probable?
a. Mieloma múltiple
b. Insuficiencia renal crónica
c. Cáncer de colon
d. Carcinoma de próstata
e. Lupus eritematoso sistémico
Mieloma múltiple: características clínicas, manifestaciones de laboratorio y
diagnóstico (uptodate)
• La anemia con un volumen corpuscular medio (VCM) normal puede
observarse con mayor probabilidad en caso de:
a. Deficiencia crónica de fierro
b. Deficiencia crónica de vitamina B12
c. Hemólisis
d. Anemia perniciosa
e. Intoxicación con plomo
• La anemia con un volumen corpuscular medio (VCM) normal puede
observarse con mayor probabilidad en caso de:
a. Deficiencia crónica de fierro
b. Deficiencia crónica de vitamina B12
c. Hemólisis
d. Anemia perniciosa
e. Intoxicación con plomo
Abordaje diagnóstico de la anemia en adultos (uptodate)
• Paciente de 50 años con diagnóstico de trombosis venosa profunda en
tratamiento con dicumarínicos. ¿Cuál de las siguientes pruebas mide la
actividad del anticoagulante?
a. Tiempo de protrombina
b. Tiempo de trombina
c. Tiempo de tromboplastina parcial activado
d. Tiempo de sangría
e. Fibrinógeno
• Paciente de 50 años con diagnóstico de trombosis venosa profunda en
tratamiento con dicumarínicos. ¿Cuál de las siguientes pruebas mide la
actividad del anticoagulante?
a. Tiempo de protrombina
b. Tiempo de trombina
c. Tiempo de tromboplastina parcial activado
d. Tiempo de sangría
e. Fibrinógeno
Uso clínico de las pruebas de coagulación (uptodate)
• Paciente obeso con edema y dolor de la pierna derecha. Examen físico:
Homans positivo. ¿Cuál es la medida inicial?
a. Warfarina
b. Aspirina
c. Enoxaparina
d. Clopidogrel
e. Prasugrel
• Paciente obeso con edema y dolor de la pierna derecha. Examen físico:
Homans positivo. ¿Cuál es la medida inicial?
a. Warfarina
b. Aspirina
c. Enoxaparina
d. Clopidogrel
e. Prasugrel
Presentación clínica y diagnóstico del adulto no embarazada con sospecha de
trombosis venosa profunda de la extremidad inferior (uptodate)
Tromboembolismo venoso: inicio de anticoagulación (uptodate)
• Tipo de anemia en pérdida sanguínea aguda:
a. Microcítica hipocrómica
b. Normocítica normocrómica
c. Microcítica normocrómica
d. Normocítica hipocrómica
e. Macrocítica hipercrómica
• Tipo de anemia en pérdida sanguínea aguda:
a. Microcítica hipocrómica
b. Normocítica normocrómica
c. Microcítica normocrómica
d. Normocítica hipocrómica
e. Macrocítica hipercrómica
Abordaje diagnóstico de la anemia en adultos (uptodate)
• Paciente de 70 años, con anemia crónica severa. VCM: 101 fL Y HCM:
25 pg. Considerando estos valores, ¿cómo clasifica a la anemia?
a. Macrocítica hipocrómica
b. Microcítica hipocrómica
c. Normocítica normocrómica
d. Macrocítica normocrómica
e. Microcítica normocrómica crónica
• Paciente de 70 años, con anemia crónica severa. VCM: 101 fL Y HCM:
25 pg. Considerando estos valores, ¿cómo clasifica a la anemia?
a. Macrocítica hipocrómica
b. Microcítica hipocrómica
c. Normocítica normocrómica
d. Macrocítica normocrómica
e. Microcítica normocrómica crónica
Abordaje diagnóstico de la anemia en adultos (uptodate)
• ¿Qué examen de laboratorio confirma el diagnóstico de anemia por
deficiencia de hierro?
a. Incremento de los niveles de ferritina
b. Capacidad de fijación de hierro disminuido
c. Disminución de receptores de transferrina
d. Disminución de los niveles de ferritina en suero
e. Saturación de transferrina aumentada.
• ¿Qué examen de laboratorio confirma el diagnóstico de anemia por
deficiencia de hierro?
a. Incremento de los niveles de ferritina
b. Capacidad de fijación de hierro disminuido
c. Disminución de receptores de transferrina
d. Disminución de los niveles de ferritina en suero
e. Saturación de transferrina aumentada.
Causas y diagnóstico de la deficiencia de hierro y la anemia ferropénica en
adultos (uptodate)
• Mujer de 65 años, que consulta por palidez, fatiga y palpitaciones al
esfuerzo desde hace dos meses. Glicemia: 98 mg%, creatinina, 1,6
mg%, hematocrito 21%, VCM de 68 fL, HCM de 21 pg/célula, CHCM de
27 g/dL, amplitud de distribución eritrocitaria (RDW) 21%. ¿Cuál tipo de
anemia es más probable?
• Refractaria
• Aplásica
• Perniciosa
• Por insuficiencia renal crónica
• Ferropénica
• Mujer de 65 años, que consulta por palidez, fatiga y palpitaciones al
esfuerzo desde hace dos meses. Glicemia: 98 mg%, creatinina, 1,6
mg%, hematocrito 21%, VCM de 68 fL, HCM de 21 pg/célula, CHCM de
27 g/dL, amplitud de distribución eritrocitaria (RDW) 21%. ¿Cuál tipo de
anemia es más probable?
• Refractaria
• Aplásica
• Perniciosa
• Por insuficiencia renal crónica
• Ferropénica
Causas y diagnóstico de la deficiencia de hierro y la anemia ferropénica en
adultos (uptodate)
• Varón de 16 años de raza negra acude por consultorio externo debido a
palidez y dolores óseos. En el examen físico presenta esplenomegalia.
Hemoglobina 8.gr/dl. ¿Qué tipo de anemia es probable?
a. Perniciosa
b. Aplásica
c. Drepanocítica
d. Déficit de folato
e. Ferropénica
• Varón de 16 años de raza negra acude por consultorio externo debido a
palidez y dolores óseos. En el examen físico presenta esplenomegalia.
Hemoglobina 8.gr/dl. ¿Qué tipo de anemia es probable?
a. Perniciosa
b. Aplásica
c. Drepanocítica
d. Déficit de folato
e. Ferropénica
Advances in the diagnosis and treatment of sickle cell diseasevances in the diagnosis and treatment of sickle
cell disease https://www.ncbi.nlm.nih.gov/pmc/articles/PMC8895633/pdf/13045_2022_Article_1237.pdf
• Varón de 33 años, con palidez, ictericia, coluria, dolor abdominal difuso
a la palpación, fiebre, taquicardia y acrocianosis por frío. Exámenes de
laboratorio: hemoglobina 8 mg/dL, reticulocitosis, Coombs directo
positivo. Frotis de sangre periférica: esferocitosis y policromía. El
tratamiento inmediato más adecuado es:
a. Transfusión de plaquetas
b. Esplenectomía
c. Transfusión de sangre
d. Glucocorticoides
e. Transfusión de plasma fresco
• Varón de 33 años, con palidez, ictericia, coluria, dolor abdominal difuso
a la palpación, fiebre, taquicardia y acrocianosis por frío. Exámenes de
laboratorio: hemoglobina 8 mg/dL, reticulocitosis, Coombs directo
positivo. Frotis de sangre periférica: esferocitosis y policromía. El
tratamiento inmediato más adecuado es:
a. Transfusión de plaquetas
b. Esplenectomía
c. Transfusión de sangre
d. Glucocorticoides
e. Transfusión de plasma fresco
Diagnóstico de anemia hemolítica en adultos (uptodate)
Anemia hemolítica autoinmune cálida (AIHA) en adultos (uptodate)
• Paciente con TEC, se encuentra inconsciente y se sospecha hemorragia
subaracnoidea. Familiar afirma que el paciente es HEMOFÍLICO. En el
perfil de coagulación, ¿qué prueba estará alterada?:
a. Tiempo parcial de tromboplastina
b. Tiempo de trombina
c. Tiempo de protrombina
d. Tiempo de sangría
e. Tiempo de lisis en globulina
• Paciente con TEC, se encuentra inconsciente y se sospecha hemorragia
subaracnoidea. Familiar afirma que el paciente es HEMOFÍLICO. En el
perfil de coagulación, ¿qué prueba estará alterada?:
a. Tiempo parcial de tromboplastina
b. Tiempo de trombina
c. Tiempo de protrombina
d. Tiempo de sangría
e. Tiempo de lisis en globulina
Manifestaciones clínicas y diagnóstico de hemofilia (uptodate)
• En un paciente que desde muy temprana edad presenta sangrado
excesivo o anormal luego de traumatismo, leve invalidez por artropatía
de rodillas y dosaje de FACTOR VIII de 3%, se diagnostica:
a. Enfermedad de von Willebrand
b. Hemofilia B
c. Hemofilia A
d. Púrpura Trombocitopénica
e. Tromboastenia
• En un paciente que desde muy temprana edad presenta sangrado
excesivo o anormal luego de traumatismo, leve invalidez por artropatía
de rodillas y dosaje de FACTOR VIII de 3%, se diagnostica:
a. Enfermedad de von Willebrand
b. Hemofilia B
c. Hemofilia A
d. Púrpura Trombocitopénica
e. Tromboastenia
Manifestaciones clínicas y diagnóstico de hemofilia (uptodate)
• Varón de 60 años. En la radiografía de cráneo presenta lesiones
osteolíticas en sacabocado; el diagnóstico más probable es:
a. Hemangioma
b. Metástasis de carcinoma
c. Histiocitosis X
d. Leucemia
e. Mieloma múltiple
• Varón de 60 años. En la radiografía de cráneo presenta lesiones
osteolíticas en sacabocado; el diagnóstico más probable es:
a. Hemangioma
b. Metástasis de carcinoma
c. Histiocitosis X
d. Leucemia
e. Mieloma múltiple
Mieloma múltiple: características clínicas, manifestaciones de laboratorio y
diagnóstico (uptodate)
• Mujer de 24 años con un mes de cansancio fácil, palidez abundantes
petequias, ginecorragia y artralgias. Pulso: 88puls/min PA 120/50mmHg
Afebril, no adenopatías, no visceromegalia. Hematocrito 24 %,
reticulocitos 3 %, leucocitos 2,100/mm3 (segmentados 15 %, eosinófilos
1 %, monocitos 4 % linfocitos 80 %), PLAQUETAS: 7,000/mm3 La
paciente tiene:
a. Anemia por deficiencia de eritropoyetina
b. Anemia hemolítica por el nivel de reticulocitos
c. Linfocitosis absoluta
d. Plaquetopenia severa y puede hacer sangrado espontáneo
e. Anemia severa y debe transfundirse inmediatamente una unidad de glóbulos rojos
• Mujer de 24 años con un mes de cansancio fácil, palidez abundantes
petequias, ginecorragia y artralgias. Pulso: 88puls/min PA 120/50mmHg
Afebril, no adenopatías, no visceromegalia. Hematocrito 24 %,
reticulocitos 3 %, leucocitos 2,100/mm3 (segmentados 15 %, eosinófilos
1 %, monocitos 4 % linfocitos 80 %), PLAQUETAS: 7,000/mm3 La
paciente tiene:
a. Anemia por deficiencia de eritropoyetina
b. Anemia hemolítica por el nivel de reticulocitos
c. Linfocitosis absoluta
d. Plaquetopenia severa y puede hacer sangrado espontáneo
e. Anemia severa y debe transfundirse inmediatamente una unidad de glóbulos rojos
Transfusión de plaquetas: indicaciones, pedidos y riesgos
asociados (uptodate)
• Paciente con manifestación de SANGRADO GINGIVAL moderado.
Tiempo de sangrado prolongado, tiempo de coagulación normal. ¿Cuál
es la deficiencia que sospecha?
a. Factor antihemofílico A
b. Vitamina K
c. Plaquetas
d. Factor Hageman
e. Fibrinógeno
• Paciente con manifestación de SANGRADO GINGIVAL moderado.
Tiempo de sangrado prolongado, tiempo de coagulación normal. ¿Cuál
es la deficiencia que sospecha?
a. Factor antihemofílico A
b. Vitamina K
c. Plaquetas
d. Factor Hageman
e. Fibrinógeno
Abordaje del adulto con sospecha de trastorno hemorrágico (uptodate)
• Varón de 26 años, que es llevado a emergencia por desorientación
fluctuante y palidez reciente. Examen de laboratorio: Hb 7 gr / dL
,PLAQUETAS 70,000 mm3, RETICULOCITOS 6 %, esquistocitos (++),
en el frotis de sangre periférica. Test de Coombs directo negativo. ¿Cuál
es el diagnóstico más probable?
a. Linfoma no Hodgkin
b. Púrpura trombótica trombocitopénica
c. Leucemia linfocítica aguda
d. Macroglobulinemia de Waldenstrom
e. Aplasia medular
• Varón de 26 años, que es llevado a emergencia por desorientación
fluctuante y palidez reciente. Examen de laboratorio: Hb 7 gr / dL
,PLAQUETAS 70,000 mm3, RETICULOCITOS 6 %, esquistocitos (++),
en el frotis de sangre periférica. Test de Coombs directo negativo. ¿Cuál
es el diagnóstico más probable?
a. Linfoma no Hodgkin
b. Púrpura trombótica trombocitopénica
c. Leucemia linfocítica aguda
d. Macroglobulinemia de Waldenstrom
e. Aplasia medular
Enfoque diagnóstico ante la sospecha de PTT, SUH u otra microangiopatía
trombótica (MAT) (uptodate)
• En adolescentes o adultos jóvenes, la causa más común de
pancitopenia es:
a. Anemia ferropénica
b. Anemia de Fanconi
c. Anemia aplásica
d. Anemia por deficiencia de cobalamina
e. Anemia por deficiencia de folato
• En adolescentes o adultos jóvenes, la causa más común de
pancitopenia es:
a. Anemia ferropénica
b. Anemia de Fanconi
c. Anemia aplásica
d. Anemia por deficiencia de cobalamina
e. Anemia por deficiencia de folato
Anemia aplásica: patogenia, manifestaciones clínicas y diagnóstico (uptodate)
• Varón de 60 años con dolores óseos, signos de compresión medular
espinal, hipercalcemia secundaria, fracturas patológicas y síndrome de
hiperviscosidad ¿Cuál es su mayor probabilidad diagnóstica?
a. Aplasia medular
b. Mieloma múltiple
c. Macroglobulinemia de Waldenstrom
d. Cáncer de próstata
e. Enfermedad por cadenas pesadas
• Varón de 60 años con dolores óseos, signos de compresión medular
espinal, hipercalcemia secundaria, fracturas patológicas y síndrome de
hiperviscosidad ¿Cuál es su mayor probabilidad diagnóstica?
a. Aplasia medular
b. Mieloma múltiple
c. Macroglobulinemia de Waldenstrom
d. Cáncer de próstata
e. Enfermedad por cadenas pesadas
Epidemiología, patogénesis, manifestaciones clínicas y diagnóstico de la
macroglobulinemia de Waldenström (uptodate)
• ¿Cuál de las siguientes alternativas corresponde al metabolismo del
hierro?
a. El ácido ascórbico inhibe la absorción intestinal
b. Los fitatos favorecen su absorción
c. El valor de ferritina representa el depósito
d. Se deposita como transferrina
e. El 90% de lo ingerido se absorbe a nivel intestinal
• ¿Cuál de las siguientes alternativas corresponde al metabolismo del
hierro?
a. El ácido ascórbico inhibe la absorción intestinal
b. Los fitatos favorecen su absorción
c. El valor de ferritina representa el depósito
d. Se deposita como transferrina
e. El 90% de lo ingerido se absorbe a nivel intestinal
Anemia por deficiencia de hierro y otras anemias microcíticas
Pregrado de hematología 4ta edición
• Paciente con anemia macrocítica crónica, cuya biopsia gástrica muestra
gastritis crónica atrófica. La causa probable de la anemia se debe a una
disminución del número de células:
a. Enterocitos
b. Principales
c. Argentafines
d. Cimógenas
e. Parietales
• Paciente con anemia macrocítica crónica, cuya biopsia gástrica muestra
gastritis crónica atrófica. La causa probable de la anemia se debe a una
disminución del número de células:
a. Enterocitos
b. Principales
c. Argentafines
d. Cimógenas
e. Parietales
Lange de Fisiología. Fisiología Gastrointestinal (1st edition)
Secreción gástrica
• Mujer que presenta palidez, fatiga, debilidad y acostumbra a comer hielo
y cal. En el examen físico se encuentra coiloniquia ¿Cuál es el
diagnóstico más probable?
a. Anemia por enfermedad crónica
b. Anemia hemolítica
c. Anemia megaloblástica
d. Anemia sideroblástica
e. Anemia ferropénica
• Mujer que presenta palidez, fatiga, debilidad y acostumbra a comer hielo
y cal. En el examen físico se encuentra coiloniquia ¿Cuál es el
diagnóstico más probable?
a. Anemia por enfermedad crónica
b. Anemia hemolítica
c. Anemia megaloblástica
d. Anemia sideroblástica
e. Anemia ferropénica
Anemia por deficiencia de hierro y otras anemias microcíticas
Pregrado de hematología 4ta edición
• ¿Cuáles son las características de la anemia ferropénica?:
a. Macrocítica hipocrómica
b. Microcítica hipocrómica
c. Microcítica normocrómica
d. Macrocítica hipercrómica
e. Normocítica normocrómica
• ¿Cuáles son las características de la anemia ferropénica?:
a. Macrocítica hipocrómica
b. Microcítica hipocrómica
c. Microcítica normocrómica
d. Macrocítica hipercrómica
e. Normocítica normocrómica
• Lesión medular en la anemia aplásica:
a. Hipocelularidad con aumento de megaloblastos
b. Hipoplasia medular y aumento de adipocitos
c. Hipercelularidad con aumento de los megaloblastos
d. Normocelularidad con aumento de adipocitos
e. Hipercelularidad con aumento de adipocitos
• Lesión medular en la anemia aplásica:
a. Hipocelularidad con aumento de megaloblastos
b. Hipoplasia medular y aumento de adipocitos
c. Hipercelularidad con aumento de los megaloblastos
d. Normocelularidad con aumento de adipocitos
e. Hipercelularidad con aumento de adipocitos
Insuficiencias medulares. Aplasia medular
Pregrado de Hematología 4ta edición
• Mujer de 30 años asintomática con Hb 8g% ¿Cuál es el tratamiento más
adecuado?
a. Ácido fólico por vía oral
b. Vitamina B12 por vía oral
c. Vitamina K por vía oral
d. Vitamina B12 por vía parenteral
e. Hierro por vía oral
• Mujer de 30 años asintomática con Hb 8g% ¿Cuál es el tratamiento más
adecuado?
a. Ácido fólico por vía oral
b. Vitamina B12 por vía oral
c. Vitamina K por vía oral
d. Vitamina B12 por vía parenteral
e. Hierro por vía oral
• Paciente con fiebre, ictericia y trastorno del sensorio, en el examen físico
se encuentran lesiones purpúricas. Exámenes auxiliares: hemoglobina:
8 gr/dl, creatinina alta 4mg/dl ¿Cuál es el diagnóstico más probable?
a. Insuficiencia renal aguda
b. Púrpura trombótica trombocitopénica
c. Púrpura trombocitopénica idiopática
d. Vasculitis
e. Lupus eritematoso sistémico
• Paciente con fiebre, ictericia y trastorno del sensorio, en el examen físico
se encuentran lesiones purpúricas. Exámenes auxiliares: hemoglobina:
8 gr/dl, creatinina alta 4mg/dl ¿Cuál es el diagnóstico más probable?
a. Insuficiencia renal aguda
b. Púrpura trombótica trombocitopénica
c. Púrpura trombocitopénica idiopática
d. Vasculitis
e. Lupus eritematoso sistémico
• ¿Cuál es la anemia que se presenta con diarrea, dispepsia y
parestesias?
a. Anemia ferropénica
b. Anemia hemolítica
c. Anemia aplásica
d. Anemia megaloblástica
e. Anemia refractaria
• ¿Cuál es la anemia que se presenta con diarrea, dispepsia y
parestesias?
a. Anemia ferropénica
b. Anemia hemolítica
c. Anemia aplásica
d. Anemia megaloblástica
e. Anemia refractaria
• La morfología de los hematíes en la anemia perniciosa es:
a. Esferocítica
b. Macrocítica
c. Normocítica
d. Microcítica
e. Drepanocítica
• La morfología de los hematíes en la anemia perniciosa es:
a. Esferocítica
b. Macrocítica
c. Normocítica
d. Microcítica
e. Drepanocítica
• Mujer 68 años, sin aparentes factores de riesgo cardiovascular, ingresa
en la unidad coronaria por un cuadro agudo de cardiopatía isquémica.
En la analítica realizada a su llegada se observa una hemoglobina de
8g/dl previamente no conocido. En este caso, la actitud más adecuada
con respecto a la anemia es:
a. Actitud expectante ya que solo se debe transfundir la anemia sintomática
b. Transfundir hematíes
c. Transfundir sangre total
d. Instaurar tratamiento con eritropoyetina
e. Administrar hierro EV
• Mujer 68 años, sin aparentes factores de riesgo cardiovascular, ingresa
en la unidad coronaria por un cuadro agudo de cardiopatía isquémica.
En la analítica realizada a su llegada se observa una hemoglobina de
8g/dl previamente no conocido. En este caso, la actitud más adecuada
con respecto a la anemia es:
a. Actitud expectante ya que solo se debe transfundir la anemia sintomática
b. Transfundir hematíes
c. Transfundir sangre total
d. Instaurar tratamiento con eritropoyetina
e. Administrar hierro EV
Indicaciones y umbrales de hemoglobina para la transfusión de glóbulos rojos
en adultos (uptodate)
• ¿Cuál de las siguientes pruebas se indica para ver el efecto de la
warfarina?
a. TTPA
b. Tiempo de sangría
c. Tiempo de protrombina
d. Recuento de plaquetas
e. Tiempo de coagulación
• ¿Cuál de las siguientes pruebas se indica para ver el efecto de la
warfarina?
a. TTPA
b. Tiempo de sangría
c. Tiempo de protrombina
d. Recuento de plaquetas
e. Tiempo de coagulación
• ¿Cuál es la reacción adversa más frecuente en el tratamiento con
heparina?
a. Leucopenia
b. Agranulocitosis
c. Trombocitopenia
d. Anemia aplásica
e. Reacción leucemoide
• ¿Cuál es la reacción adversa más frecuente en el tratamiento con
heparina?
a. Leucopenia
b. Agranulocitosis
c. Trombocitopenia
d. Anemia aplásica
e. Reacción leucemoide
Heparina y heparina BPM: posología y efectos adversos (uptodate)
• Mujer de 39 años acudió a consulta con antecedentes de 4 años de fatiga y
disfagia que empeora progresivamente hasta sólidos. Siguió una dieta
vegetariana y no reportó molestias gastrointestinales, sangrado o menorragia.
Al examen físico se destacó palidez conjuntival y coiloniquia. Los estudios de
laboratorio revelaron anemia por deficiencia de hierro grave. Una endoscopia
superior reveló al menos dos estenosis del esófago cervical; la estenosis
ubicada más cranealmente no pudo ser pasado por el endoscopio. Un estudio
posterior con trago de bario identificó dos redes esofágicas, membranas
asimétricas que sobresalen hacia la luz del esófago, como causa de las
estenosis. El tratamiento de la paciente sería:
a. Radioterapia y quimioterapia
b. Endoligadura de várices esofágicas
c. Hierro sacarato y sulfato ferroso
d. Antiparasitarios para enfermedad de Chagas
e. Omeprazol, sucralfato y procinéticos
• Mujer de 39 años acudió a consulta con antecedentes de 4 años de fatiga y
disfagia que empeora progresivamente hasta sólidos. Siguió una dieta
vegetariana y no reportó molestias gastrointestinales, sangrado o menorragia.
Al examen físico se destacó palidez conjuntival y coiloniquia. Los estudios de
laboratorio revelaron anemia por deficiencia de hierro grave. Una endoscopia
superior reveló al menos dos estenosis del esófago cervical; la estenosis
ubicada más cranealmente no pudo ser pasado por el endoscopio. Un estudio
posterior con trago de bario identificó dos redes esofágicas, membranas
asimétricas que sobresalen hacia la luz del esófago, como causa de las
estenosis. El tratamiento de la paciente sería:
a. Radioterapia y quimioterapia
b. Endoligadura de várices esofágicas
c. Hierro sacarato y sulfato ferroso
d. Antiparasitarios para enfermedad de Chagas
e. Omeprazol, sucralfato y procinéticos
• Un hombre de 67 años presenta prurito intolerable, especialmente
cuando toma una ducha caliente. No fuma y no toma medicamentos. Es
natural y procedente de Lima, de ocupación oficinista. El valor del
hematocrito es del 60% y tiene esplenomegalia. Señale el diagnóstico
más probable:
a. Cor pulmonar
b. Coledocolitiasis
c. Cirrosis hepática
d. Policitemia vera
e. Leucemia mieloide
• Un hombre de 67 años presenta prurito intolerable, especialmente
cuando toma una ducha caliente. No fuma y no toma medicamentos. Es
natural y procedente de Lima, de ocupación oficinista. El valor del
hematocrito es del 60% y tiene esplenomegalia. Señale el diagnóstico
más probable:
a. Cor pulmonar
b. Coledocolitiasis
c. Cirrosis hepática
d. Policitemia vera
e. Leucemia mieloide
Manifestaciones clínicas y diagnóstico de policitemia vera (uptodate)
• Un hombre de 67 años es evaluado por pies enrojecidos, calientes,
dolorosos y recuento de plaquetas de 975.000/μl. El historial médico es
positivo para IMA previo. Niega sangrado reciente, anemia ferropénica o
procesos infecciosos agudos. Señale el diagnóstico más probable:
a. Eritromelalgia
b. Trombocitemia esencial
c. Síndrome de Wiskott-Aldrich
d. Deficiencia de vitamina B12
e. Síndrome mielodisplásico
• Un hombre de 67 años es evaluado por pies enrojecidos, calientes,
dolorosos y recuento de plaquetas de 975.000/μl. El historial médico es
positivo para IMA previo. Niega sangrado reciente, anemia ferropénica o
procesos infecciosos agudos. Señale el diagnóstico más probable:
a. Eritromelalgia
b. Trombocitemia esencial
c. Síndrome de Wiskott-Aldrich
d. Deficiencia de vitamina B12
e. Síndrome mielodisplásico
Manifestaciones clínicas, patogénesis y diagnóstico de la trombocitemia
esencial (uptodate)
Una mujer de 42 años es evaluada durante varios meses por fatiga. Ella refiere una
dieta normal y sin problemas gastrointestinales. síntomas. El historial médico es
notable por hipotiroidismo y vitíligo. Su única medicación es la levotiroxina. Al
examen físico los signos vitales son normales. Ella presenta varias máculas
hipopigmentadas, bien definidas, distribuidas simétricamente en las extremidades y
el tronco. Analítica: Hemoglobina 8,8 g/dl, recuento de leucocitos 4000/uL, volumen
corpuscular medio 117 fl, recuento de plaquetas 160.000/uL, reticulocitos 1%. Se
muestra un frotis de sangre periférica. ¿Cuál de las siguientes es la prueba
diagnóstica inicial más adecuada?

a. Nivel de homocisteína
b. Nivel de ácido metilmalónico
c. Nivel de folato sérico
d. Nivel sérico de vitamina B12
e. Nivel de tiamina sérica
Una mujer de 42 años es evaluada durante varios meses por fatiga. Ella refiere una
dieta normal y sin problemas gastrointestinales. síntomas. El historial médico es
notable por hipotiroidismo y vitíligo. Su única medicación es la levotiroxina. Al
examen físico los signos vitales son normales. Ella presenta varias máculas
hipopigmentadas, bien definidas, distribuidas simétricamente en las extremidades y
el tronco. Analítica: Hemoglobina 8,8 g/dl, recuento de leucocitos 4000/uL, volumen
corpuscular medio 117 fl, recuento de plaquetas 160.000/uL, reticulocitos 1%. Se
muestra un frotis de sangre periférica. ¿Cuál de las siguientes es la prueba
diagnóstica inicial más adecuada?

a. Nivel de homocisteína
b. Nivel de ácido metilmalónico
c. Nivel de folato sérico
d. Nivel sérico de vitamina B12
e. Nivel de tiamina sérica
Manifestaciones clínicas y diagnóstico de deficiencia de vitamina B12 y folato
(uptodate)
Mujer de 29 años es evaluada en urgencias por epistaxis y sangrado de encías que comenzó
hace 3 días. Por lo demás, el historial médico no tiene nada de especial. y ella no toma
medicamentos. En el examen físico, la temperatura es de 36,7 °C, la presión arterial es de
110/80 mm Hg, la frecuencia del pulso es de 120/min y la frecuencia respiratoria es de
22/min. Se ve sangre seca en la nariz y se nota sangrado gingival. No hay linfadenopatía ni
hepatoesplenomegalia. Hay petequias en las extremidades inferiores. Analítica: Hemoglobina
8,0 g/dl, recuento de leucocitos 14.000/uL (19% neutrófilos, 3% bandas. 32% linfocitos. 15%
monocitos y 31% células "atípicas"), recuento de plaquetas 8.000/uL. El tiempo de
protrombina, el tiempo de tromboplastina parcial activada y el INR están elevados; El nivel de
flbrinógeno es bajo y los productos de degradación de la fibrina están elevados. Se muestra un
frotis de sangre periférica. ¿Cuál de los siguientes es el diagnóstico más probable?

a. Leucemia linfoblástica aguda


b. Leucemia promielocítica aguda
c. Anemia aplásica
d. Mielofibrosis primaria
e. Deficiencia de vitamina B12
Mujer de 29 años es evaluada en urgencias por epistaxis y sangrado de encías que comenzó
hace 3 días. Por lo demás, el historial médico no tiene nada de especial. y ella no toma
medicamentos. En el examen físico, la temperatura es de 36,7 °C, la presión arterial es de
110/80 mm Hg, la frecuencia del pulso es de 120/min y la frecuencia respiratoria es de
22/min. Se ve sangre seca en la nariz y se nota sangrado gingival. No hay linfadenopatía ni
hepatoesplenomegalia. Hay petequias en las extremidades inferiores. Analítica: Hemoglobina
8,0 g/dl, recuento de leucocitos 14.000/uL (19% neutrófilos, 3% bandas. 32% linfocitos. 15%
monocitos y 31% células "atípicas"), recuento de plaquetas 8.000/uL. El tiempo de
protrombina, el tiempo de tromboplastina parcial activada y el INR están elevados; El nivel de
flbrinógeno es bajo y los productos de degradación de la fibrina están elevados. Se muestra un
frotis de sangre periférica. ¿Cuál de los siguientes es el diagnóstico más probable?

a. Leucemia linfoblástica aguda


b. Leucemia promielocítica aguda
c. Anemia aplásica
d. Mielofibrosis primaria
e. Deficiencia de vitamina B12
Manifestaciones clínicas, características patológicas y diagnóstico de leucemia
promielocítica aguda en adultos (uptodate)
• Una mujer de 40 años está hospitalizada con dificultad respiratoria aguda. El
historial médico es importante para dos pérdidas de embarazos en el segundo
trimestre. Las pruebas de laboratorio realizadas hace 3 meses después de su
segunda pérdida del embarazo dieron positivo para anticoagulante lúpico,
anticuerpos anticardiolipina IgG de título alto y anticuerpos IgG anti beta 2
glucoproteína de título alto. Por lo demás se encuentra bien y no toma
medicamentos. En el examen físico, la frecuencia del pulso es de 120/min y la
frecuencia respiratoria de 20/min; La saturación de oxígeno es del 92%
respirando aire ambiente. Analíticas: anticoagulante lúpico positivo, anticuerpos
anticardiolipina IgG de título alto y anticuerpos anti beta 2 glucoproteína IgG de
título alto. La angiografía por TC muestra una embolia pulmonar en la rama
principal de la arteria pulmonar derecha. Se inicia heparina de bajo peso
molecular. ¿Cuál de los siguientes es el tratamiento a largo plazo más
adecuado?
a. Dabigatrán más aspirina
b. Rivaroxabán
c. Warfarina
d. Warfarina más filtro de vena cava inferior
e. Apixabán por la insuficiencia renal
• Una mujer de 40 años está hospitalizada con dificultad respiratoria aguda. El
historial médico es importante para dos pérdidas de embarazos en el segundo
trimestre. Las pruebas de laboratorio realizadas hace 3 meses después de su
segunda pérdida del embarazo dieron positivo para anticoagulante lúpico,
anticuerpos anticardiolipina IgG de título alto y anticuerpos IgG anti beta 2
glucoproteína de título alto. Por lo demás se encuentra bien y no toma
medicamentos. En el examen físico, la frecuencia del pulso es de 120/min y la
frecuencia respiratoria de 20/min; La saturación de oxígeno es del 92%
respirando aire ambiente. Analíticas: anticoagulante lúpico positivo, anticuerpos
anticardiolipina IgG de título alto y anticuerpos anti beta 2 glucoproteína IgG de
título alto. La angiografía por TC muestra una embolia pulmonar en la rama
principal de la arteria pulmonar derecha. Se inicia heparina de bajo peso
molecular. ¿Cuál de los siguientes es el tratamiento a largo plazo más
adecuado?
a. Dabigatrán más aspirina
b. Rivaroxabán
c. Warfarina
d. Warfarina más filtro de vena cava inferior
e. Apixabán por la insuficiencia renal
Manejo del síndrome antifosfolípido (uptodate)
• Una mujer de 67 años es evaluada por fatiga y disnea de esfuerzo durante los
últimos 2 meses. Es importante el historial médico del cáncer de mama de
hace 15 años, que se trataba con cirugía, quimioterapia y radiación. Por lo
demás, su historial no tiene nada de especial y no toma medicamentos. Al
examen físico los signos vitales son normales. La saturación de oxígeno es del
97% respirando aire ambiente. Se evidencia palidez cutánea. La exploración
abdominal no revela hepatoesplenomegalia, petequias ni hematomas.
Analítica: Hemoglobina 7,7 g/dL, recuento de leucocitos 3600/uL (35%
neutrófilos, 23% linfocitos, 21% monocitos, sin blastos), volumen corpuscular
medio 110 fl, recuento de reticulocitos 1,5% de eritrocitos, recuento de
plaquetas 105.000/uL. La vitamina B 12 y el folato eran normales. ¿Cuál es la
prueba más apropiada para realizar a continuación?:
a. Biopsia de médula ósea
b. Colonoscopia
c. Prueba de antiglobulina directa
d. Niveles de hierro y ferritina y capacidad total de unión al hierro.
e. Eritropoyetina
• Una mujer de 67 años es evaluada por fatiga y disnea de esfuerzo durante los
últimos 2 meses. Es importante el historial médico del cáncer de mama de
hace 15 años, que se trataba con cirugía, quimioterapia y radiación. Por lo
demás, su historial no tiene nada de especial y no toma medicamentos. Al
examen físico los signos vitales son normales. La saturación de oxígeno es del
97% respirando aire ambiente. Se evidencia palidez cutánea. La exploración
abdominal no revela hepatoesplenomegalia, petequias ni hematomas.
Analítica: Hemoglobina 7,7 g/dL, recuento de leucocitos 3600/uL (35%
neutrófilos, 23% linfocitos, 21% monocitos, sin blastos), volumen corpuscular
medio 110 fl, recuento de reticulocitos 1,5% de eritrocitos, recuento de
plaquetas 105.000/uL. La vitamina B 12 y el folato eran normales. ¿Cuál es la
prueba más apropiada para realizar a continuación?:
a. Biopsia de médula ósea
b. Colonoscopia
c. Prueba de antiglobulina directa
d. Niveles de hierro y ferritina y capacidad total de unión al hierro.
e. Eritropoyetina
Síndromes mielodisplásicos
Pregrado de Hematología 4ta edición
• Una mujer de 32 años se evalúa por disnea de esfuerzo de 2 semanas.
Antecedente médicos de lupus eritematoso sistémico. Los antecedentes
familiares no tienen nada de especial. Su única medicación es
hidroxicloroquina. En el examen físico, la presión arterial es de 98/60 mm Hg y
la frecuencia del pulso es de 107/min; otros signos vitales son normales. La
saturación de oxígeno es del 98% respirando aire ambiente. Aparte de la
taquicardia, el examen físico es normal. Estudios de laboratorio: Haptoglobina
indetectable, hemoglobina 9,8 g/dL, recuento de leucocitos 4200/uL, recuento
de plaquetas 125.000/uL, recuento de reticulocitos 15% de los eritrocitos,
lactato deshidrogenasa 475 U/L, frotis de sangre periférica muestra esferocitos.
¿Cuál de los siguientes es el test diagnóstico más adecuado para realizar a
continuación?:
a. Dosaje de vitamina B12 sérica
b. Biopsia de médula ósea
c. Prueba de antiglobulina directa
d. Prueba de fragilidad osmótica
e. Electroforesis de hemoglobina
• Una mujer de 32 años se evalúa por disnea de esfuerzo de 2 semanas.
Antecedente médicos de lupus eritematoso sistémico. Los antecedentes
familiares no tienen nada de especial. Su única medicación es
hidroxicloroquina. En el examen físico, la presión arterial es de 98/60 mm Hg y
la frecuencia del pulso es de 107/min; otros signos vitales son normales. La
saturación de oxígeno es del 98% respirando aire ambiente. Aparte de la
taquicardia, el examen físico es normal. Estudios de laboratorio: Haptoglobina
indetectable, hemoglobina 9,8 g/dL, recuento de leucocitos 4200/uL, recuento
de plaquetas 125.000/uL, recuento de reticulocitos 15% de los eritrocitos,
lactato deshidrogenasa 475 U/L, frotis de sangre periférica muestra esferocitos.
¿Cuál de los siguientes es el test diagnóstico más adecuado para realizar a
continuación?:
a. Dosaje de vitamina B12 sérica
b. Biopsia de médula ósea
c. Prueba de antiglobulina directa
d. Prueba de fragilidad osmótica
e. Electroforesis de hemoglobina
Diagnóstico de anemia hemolítica en adultos (uptodate)
• Una mujer de 25 años es hospitalizada por hemorragia un día después
de una amigdalectomía. Refiere antecedentes de menstruaciones
abundantes desde la menarquia, pero sin hematomas espontáneos ni
hematomas musculares o articulares. En el examen físico, los signos
vitales y los hallazgos de la exploración son normales. Estudios de
laboratorio: Tiempo de tromboplastina parcial activada (aPTT) 37 s,
aPTT con estudio de mezcla 26 s, hematocrito 30%, recuento de
plaquetas 190.000/L, tiempo de protrombina 11 s, prueba de función
plaquetaria normal. ¿Cuál de las siguientes es la prueba diagnóstica
más adecuada para realizar a continuación?
a. Prueba del veneno de víbora de Russell
b. Medir los niveles de factores
c. Estudio de agregación plaquetaria
d. Ensayo de antígeno de von Willebrand
e. Prueba del lazo de Rumpel Leede
• Una mujer de 25 años es hospitalizada por hemorragia un día después
de una amigdalectomía. Refiere antecedentes de menstruaciones
abundantes desde la menarquia, pero sin hematomas espontáneos ni
hematomas musculares o articulares. En el examen físico, los signos
vitales y los hallazgos de la exploración son normales. Estudios de
laboratorio: Tiempo de tromboplastina parcial activada (aPTT) 37 s,
aPTT con estudio de mezcla 26 s, hematocrito 30%, recuento de
plaquetas 190.000/L, tiempo de protrombina 11 s, prueba de función
plaquetaria normal. ¿Cuál de las siguientes es la prueba diagnóstica
más adecuada para realizar a continuación?
a. Prueba del veneno de víbora de Russell
b. Medir los niveles de factores
c. Estudio de agregación plaquetaria
d. Ensayo de antígeno de von Willebrand
e. Prueba del lazo de Rumpel Leede
Deficiencia del factor XI (once) (uptodate)
Deficiencia del factor XI (once) (uptodate)
• Un hombre de 45 años es evaluado por un desorden cerebrovascular agudo. Fue
hospitalizado hace 48 horas tras sufrir un accidente automovilístico en el que sufrió
varias fracturas que requirieron fijación interna. Hace 24 horas le diagnosticaron una
embolia pulmonar y le iniciaron infusión de heparina no fraccionada. Ha desarrollado
síntomas neurológicos durante los últimos 30 minutos. Antes de la hospitalización, no
tomaba medicamentos. En el examen físico, la temperatura es normal, la presión
arterial es 110/60 mm Hg, la frecuencia del pulso es de 100/min y la frecuencia
respiratoria es de 16/min. La saturación de oxígeno es del 94%, respirando oxígeno, 2
L/min. El paciente está despierto con función del lenguaje normal. No hay papiledema.
Las pupilas son isocóricas y reactivas. Se observa debilidad facial izquierda, disartria y
parálisis fláccida en el brazo izquierdo. La tomografía computarizada sin contraste
muestra una hemorragia intracerebral aguda, de 1 cm de diámetro en los ganglios
basales derechos, sin hemorragia intraventricular ni desplazamiento de la línea media.
Se suspende la heparina no fraccionada. ¿Cuál de los siguientes es el tratamiento
adicional más adecuado?
a. Crioprecipitado
b. ldarucizumab
c. Sulfato de protamina
d. Vitamina K
e. Plasma fresco congelado
• Un hombre de 45 años es evaluado por un desorden cerebrovascular agudo. Fue
hospitalizado hace 48 horas tras sufrir un accidente automovilístico en el que sufrió
varias fracturas que requirieron fijación interna. Hace 24 horas le diagnosticaron una
embolia pulmonar y le iniciaron infusión de heparina no fraccionada. Ha desarrollado
síntomas neurológicos durante los últimos 30 minutos. Antes de la hospitalización, no
tomaba medicamentos. En el examen físico, la temperatura es normal, la presión
arterial es 110/60 mm Hg, la frecuencia del pulso es de 100/min y la frecuencia
respiratoria es de 16/min. La saturación de oxígeno es del 94%, respirando oxígeno, 2
L/min. El paciente está despierto con función del lenguaje normal. No hay papiledema.
Las pupilas son isocóricas y reactivas. Se observa debilidad facial izquierda, disartria y
parálisis fláccida en el brazo izquierdo. La tomografía computarizada sin contraste
muestra una hemorragia intracerebral aguda, de 1 cm de diámetro en los ganglios
basales derechos, sin hemorragia intraventricular ni desplazamiento de la línea media.
Se suspende la heparina no fraccionada. ¿Cuál de los siguientes es el tratamiento
adicional más adecuado?
a. Crioprecipitado
b. ldarucizumab
c. Sulfato de protamina
d. Vitamina K
e. Plasma fresco congelado
Reversión de la anticoagulación en hemorragia intracraneal (uptodate)
Reversión de la anticoagulación en hemorragia intracraneal (uptodate)
Una mujer de 32 años es visitada para evaluación de seguimiento por anemia crónica
que se ha mantenido estable durante al menos los últimos 5 años. No tiene
antecedentes de transfusiones. Tuvo un embarazo a los 29 años, que terminó con
muerte fetal intrauterina a principios del tercer trimestre. Su padre tiene anemia. El
paciente es chino. En la exploración física, los signos vitales y el resto de la
exploración física son normales. Estudios de laboratorio: Hemoglobina 10,8 g/dl,
volumen corpuscular medio 62 fl, recuento de reticulocitos 2% de los eritrocitos.
Estudios de hierro: ferritina 200 ng/ml, hierro 200 ug/dL, capacidad total de fijación
de hierro 280 pg/dl, electroforesis de hemoglobina revela un patrón de migración
normal de hemoglobina A y niveles normales de hemoglobina A y hemoglobina F. Se
muestra un frotis de sangre periférica. ¿Cuál de los siguientes es el diagnóstico más
probable?

a. Portador de alfa talasemia


b. Rasgo de alfa talasemia
c. Beta talasemia menor
d. Rasgo de hemoglobina E
e. Anemia ferropénica
Una mujer de 32 años es visitada para evaluación de seguimiento por anemia crónica
que se ha mantenido estable durante al menos los últimos 5 años. No tiene
antecedentes de transfusiones. Tuvo un embarazo a los 29 años, que terminó con
muerte fetal intrauterina a principios del tercer trimestre. Su padre tiene anemia. El
paciente es chino. En la exploración física, los signos vitales y el resto de la
exploración física son normales. Estudios de laboratorio: Hemoglobina 10,8 g/dl,
volumen corpuscular medio 62 fl, recuento de reticulocitos 2% de los eritrocitos.
Estudios de hierro: ferritina 200 ng/ml, hierro 200 ug/dL, capacidad total de fijación
de hierro 280 pg/dl, electroforesis de hemoglobina revela un patrón de migración
normal de hemoglobina A y niveles normales de hemoglobina A y hemoglobina F. Se
muestra un frotis de sangre periférica. ¿Cuál de los siguientes es el diagnóstico más
probable?

a. Portador de alfa talasemia


b. Rasgo de alfa talasemia
c. Beta talasemia menor
d. Rasgo de hemoglobina E
e. Anemia ferropénica
Diagnóstico de talasemia (adultos y niños) (uptodate)
• Una mujer de 19 años es evaluada tras un diagnóstico reciente de
esferocitosis hereditaria. Ella es asintomática. Ella no toma
medicamentos. Al examen físico los signos vitales son normales. La
punta del bazo es palpable. Los estudios de laboratorio muestran un
nivel de hemoglobina de 11,7 g/dl, una concentración media de
hemoglobina corpuscular de 40 g/dL y un recuento de reticulocitos del
6% de los eritrocitos. El frotis de sangre periférica muestra frecuentes
esferocitos y policromasia de los eritrocitos y reticulocitosis. ¿Cuál de los
siguientes es el tratamiento más adecuado?
a. Folato
b. Prednisona
c. Rituximab
d. Esplenectomía
e. Hidroxicobalamina
• Una mujer de 19 años es evaluada tras un diagnóstico reciente de
esferocitosis hereditaria. Ella es asintomática. Ella no toma
medicamentos. Al examen físico los signos vitales son normales. La
punta del bazo es palpable. Los estudios de laboratorio muestran un
nivel de hemoglobina de 11,7 g/dl, una concentración media de
hemoglobina corpuscular de 40 g/dL y un recuento de reticulocitos del
6% de los eritrocitos. El frotis de sangre periférica muestra frecuentes
esferocitos y policromasia de los eritrocitos y reticulocitosis. ¿Cuál de los
siguientes es el tratamiento más adecuado?
a. Folato
b. Prednisona
c. Rituximab
d. Esplenectomía
e. Hidroxicobalamina
Esferocitosis hereditaria (uptodate)
• Una mujer de 28 años acude a evaluación por aparición de hematomas,
epistaxis y sangrado de encías con facilidad. Tiene 16 semanas de
gestación. Los medicamentos incluyen sólo vitaminas prenatales. Al
examen físico los signos vitales son normales. Se nota sangre seca
alrededor de las fosas nasales. Tiene un útero grávido. Estudios de
laboratorio: Hemoglobina 12,2 g/dL, leucocitos 4500/uL, plaquetas
66.000/uL, creatinina 0,7 mg/dl. El frotis de sangre periférica muestra
eritrocitos y granulocitos normales sin aglutinación de plaquetas. ¿Cuál
es el diagnóstico más probable de su trombocitopenia?
a. Trombocitopenia gestacional
b. Síndrome HELLP
c. Púrpura trombocitopénica inmunitaria
d. Púrpura trombocitopénica trombótica
e. Deficiencia de folato
• Una mujer de 28 años acude a evaluación por aparición de hematomas,
epistaxis y sangrado de encías con facilidad. Tiene 16 semanas de
gestación. Los medicamentos incluyen sólo vitaminas prenatales. Al
examen físico los signos vitales son normales. Se nota sangre seca
alrededor de las fosas nasales. Tiene un útero grávido. Estudios de
laboratorio: Hemoglobina 12,2 g/dL, leucocitos 4500/uL, plaquetas
66.000/uL, creatinina 0,7 mg/dl. El frotis de sangre periférica muestra
eritrocitos y granulocitos normales sin aglutinación de plaquetas. ¿Cuál
es el diagnóstico más probable de su trombocitopenia?
a. Trombocitopenia gestacional
b. Síndrome HELLP
c. Púrpura trombocitopénica inmunitaria
d. Púrpura trombocitopénica trombótica
e. Deficiencia de folato
Trombocitopenia en el embarazo (uptodate)
Trombocitopenia en el embarazo (uptodate)
• Un hombre de 49 años es evaluado por fatiga progresiva y una pérdida de peso de
13,6 kg (30 lb) en 4 meses. Su historial médico no tiene nada especial y no toma
medicamentos. Al examen físico los signos vitales son normales. En la palpación
abdominal, el bazo tiene 17 cm. Estudios de laboratorio: Hemoglobina 11 g/dL,
recuento de leucocitos 87.500/uL (87% neutrófilos, 2% bandas, 2% metamielocitos, 3%
mieloblastos, 2% basófilos, 2% linfocitos), recuento de plaquetas 465.000/uL. La
biopsia de médula ósea muestra médula hipercelular con hiperplasia granulocítica y
6% de mieloblastos. Los análisis cromosómicos y los estudios de hibridación in situ de
fluorescencia muestran t(9;22). ¿Cuál de los siguientes es el tratamiento más
adecuado?
a. Hidroxiurea
b. Imatinib
c. Quimioterapia de inducción
d. Trasplante de células madre
e. Corticoterapia
• Un hombre de 49 años es evaluado por fatiga progresiva y una pérdida de peso de
13,6 kg (30 lb) en 4 meses. Su historial médico no tiene nada especial y no toma
medicamentos. Al examen físico los signos vitales son normales. En la palpación
abdominal, el bazo tiene 17 cm. Estudios de laboratorio: Hemoglobina 11 g/dL,
recuento de leucocitos 87.500/uL (87% neutrófilos, 2% bandas, 2% metamielocitos, 3%
mieloblastos, 2% basófilos, 2% linfocitos), recuento de plaquetas 465.000/uL. La
biopsia de médula ósea muestra médula hipercelular con hiperplasia granulocítica y
6% de mieloblastos. Los análisis cromosómicos y los estudios de hibridación in situ de
fluorescencia muestran t(9;22). ¿Cuál de los siguientes es el tratamiento más
adecuado?
a. Hidroxiurea
b. Imatinib
c. Quimioterapia de inducción
d. Trasplante de células madre
e. Corticoterapia
Descripción general del tratamiento de la leucemia mieloide crónica (uptodate)
Descripción general del tratamiento de la leucemia mieloide crónica (uptodate)
• Un hombre de 68 años fue hospitalizado por hemorragia digestiva alta aguda.
Mientras recibía su primera unidad de concentrado de glóbulos rojos,
desarrolló fiebre, disnea y dolor en el costado a los 15 minutos de la
transfusión. La transfusión fue detenida por el personal de enfermería. Su
única medicación es el omeprazol. En el examen físico, la temperatura es de
38,3 °C, la presión arterial es de 80/40 mm Hg y la frecuencia del pulso es de
120/min. y la frecuencia respiratoria es de 18/min. La saturación de oxígeno es
del 96%, respirando aire ambiente. El examen cardiopulmonar revela pulmones
limpios, ruidos cardíacos normales y presión venosa central baja. La
inspección de una muestra de orina recolectada revela una coloración rosada.
¿Cuál de los siguientes es el diagnóstico más probable?
a. Reacción transfusional hemolítica aguda
b. Reacción alérgica anafiláctica
c. Sobrecarga circulatoria asociada a transfusión
d. Lesión pulmonar aguda relacionada con transfusiones
e. Rabdomiólisis relacionada con transfusiones
• Un hombre de 68 años fue hospitalizado por hemorragia digestiva alta aguda.
Mientras recibía su primera unidad de concentrado de glóbulos rojos,
desarrolló fiebre, disnea y dolor en el costado a los 15 minutos de la
transfusión. La transfusión fue detenida por el personal de enfermería. Su
única medicación es el omeprazol. En el examen físico, la temperatura es de
38,3 °C, la presión arterial es de 80/40 mm Hg y la frecuencia del pulso es de
120/min. y la frecuencia respiratoria es de 18/min. La saturación de oxígeno es
del 96%, respirando aire ambiente. El examen cardiopulmonar revela pulmones
limpios, ruidos cardíacos normales y presión venosa central baja. La
inspección de una muestra de orina recolectada revela una coloración rosada.
¿Cuál de los siguientes es el diagnóstico más probable?
a. Reacción transfusional hemolítica aguda
b. Reacción alérgica anafiláctica
c. Sobrecarga circulatoria asociada a transfusión
d. Lesión pulmonar aguda relacionada con transfusiones
e. Rabdomiólisis relacionada con transfusiones
Reacciones hemolíticas de transfusión (uptodate)
• Una mujer de 75 años es evaluada por fatiga y antecedentes de 3 meses de dolor en
cuello, hombros y caderas y fiebres bajas. No informa dolores de cabeza, cambios en
la visión ni claudicación de la mandíbula. Su última colonoscopia fue hace 5 años. No
tiene otros problemas médicos y no toma medicamentos. Al examen físico los signos
vitales son normales. No se observa dolor temporal ni en la mandíbula. Los músculos
proximales de la cintura son dolorosos a la palpación, con rango de movimiento
restringido debido a la rigidez y el dolor. La fuerza muscular es simétrica y normal. No
se evidencian derrames articulares ni sinovitis. Estudios de laboratorio: velocidad de
sedimentación globular 90 mm/h, hemoglobina 8,5 g/dl, recuento de leucocitos
8.000/uL con diferencial normal, recuento de plaquetas 450.000/uL, volumen
corpuscular medio 84 fL, recuento de reticulocitos 1% de los eritrocitos. Estudios de
hierro: ferritina 800 ng/ml, hierro 10 pg/ml, capacidad total de fijación de hierro 200
pg/dl. ¿Cuál de los siguientes es el manejo más adecuado de la anemia de este
paciente?
a. Biopsia de médula ósea
b. Colonoscopia
c. Agente estimulante de la eritropoyesis
d. Reemplazo de hierro oral
e. Prednisona en dosis bajas
• Una mujer de 75 años es evaluada por fatiga y antecedentes de 3 meses de dolor en
cuello, hombros y caderas y fiebres bajas. No informa dolores de cabeza, cambios en
la visión ni claudicación de la mandíbula. Su última colonoscopia fue hace 5 años. No
tiene otros problemas médicos y no toma medicamentos. Al examen físico los signos
vitales son normales. No se observa dolor temporal ni en la mandíbula. Los músculos
proximales de la cintura son dolorosos a la palpación, con rango de movimiento
restringido debido a la rigidez y el dolor. La fuerza muscular es simétrica y normal. No
se evidencian derrames articulares ni sinovitis. Estudios de laboratorio: velocidad de
sedimentación globular 90 mm/h, hemoglobina 8,5 g/dl, recuento de leucocitos
8.000/uL con diferencial normal, recuento de plaquetas 450.000/uL, volumen
corpuscular medio 84 fL, recuento de reticulocitos 1% de los eritrocitos. Estudios de
hierro: ferritina 800 ng/ml, hierro 10 pg/ml, capacidad total de fijación de hierro 200
pg/dl. ¿Cuál de los siguientes es el manejo más adecuado de la anemia de este
paciente?
a. Biopsia de médula ósea
b. Colonoscopia
c. Agente estimulante de la eritropoyesis
d. Reemplazo de hierro oral
e. Prednisona en dosis bajas
Anemia de enfermedad crónica/anemia de inflamación (uptodate)
Anemia de enfermedad crónica/anemia de inflamación (uptodate)
Manifestaciones clínicas y diagnóstico de polimialgia reumática (uptodate)
Una mujer de 19 años es evaluada por hematomas fáciles de 2 semanas
de duración. No presenta otros síntomas y su historial médico no tiene
nada de especial. Ella no toma medicamentos. Al examen físico los signos
vitales son normales. Los hallazgos de la exploración se limitan a
petequias en las extremidades inferiores y pequeñas equimosis dispersas.
Los estudios de laboratorio muestran un recuento de plaquetas de
15.000/uL; el resto del hemograma completo es normal. Se muestra el
frotis de sangre periférica. Las pruebas de VIH y hepatitis C están
pendientes. ¿Cuál de los siguientes es el manejo más adecuado?

a. Glucocorticoides
b. Intercambio de plasma
c. Transfusión de plaquetas
d. Observación
e. Esplenectomía
Una mujer de 19 años es evaluada por hematomas fáciles de 2 semanas
de duración. No presenta otros síntomas y su historial médico no tiene
nada de especial. Ella no toma medicamentos. Al examen físico los signos
vitales son normales. Los hallazgos de la exploración se limitan a
petequias en las extremidades inferiores y pequeñas equimosis dispersas.
Los estudios de laboratorio muestran un recuento de plaquetas de
15.000/uL; el resto del hemograma completo es normal. Se muestra el
frotis de sangre periférica. Las pruebas de VIH y hepatitis C están
pendientes. ¿Cuál de los siguientes es el manejo más adecuado?

a. Glucocorticoides
b. Intercambio de plasma
c. Transfusión de plaquetas
d. Observación
e. Esplenectomía
Tratamiento inicial de la trombocitopenia inmune (PTI) en adultos (uptodate)
• Un hombre de 32 años es evaluado en el servicio de urgencias por
antecedentes de fatiga, febrícula, hematomas y epistaxis de una semana de
evolución. No tiene otros problemas médicos y no toma medicamentos. En el
examen físico, la temperatura es de 37,8 °C, la presión arterial es de 120/65
mm Hg, la frecuencia del pulso es de 108/min y la frecuencia respiratoria es de
22/min. Se nota palidez. Hay sangre seca en las fosas nasales. Se observan
múltiples hematomas en sus extremidades y petequias en los tobillos. Estudios
de laboratorio: Tiempo de tromboplastina parcial activada 38 seg, dímero D 2,5
mg/L, hematocrito 22%, recuento de leucocitos 2.300/uL, recuento de
plaquetas 22.000/uL, tiempo de protrombina 25 seg, albúmina 3,5 g/dL, alanina
aminotransferasa 30 U /L, aspartato aminotransferasa 35 U/L, bilirrubina total
0,9 mg/dL, fibrinógeno 60 mg/dL. ¿Cuál de los siguientes es el diagnóstico más
probable?
a. Anemia aplásica
b. Coagulopatía de la enfermedad hepática.
c. Coagulación intravascular diseminada
d. Púrpura trombocitopénica trombótica
e. Síndrome de Wiskott Aldrich
• Un hombre de 32 años es evaluado en el servicio de urgencias por
antecedentes de fatiga, febrícula, hematomas y epistaxis de una semana de
evolución. No tiene otros problemas médicos y no toma medicamentos. En el
examen físico, la temperatura es de 37,8 °C, la presión arterial es de 120/65
mm Hg, la frecuencia del pulso es de 108/min y la frecuencia respiratoria es de
22/min. Se nota palidez. Hay sangre seca en las fosas nasales. Se observan
múltiples hematomas en sus extremidades y petequias en los tobillos. Estudios
de laboratorio: Tiempo de tromboplastina parcial activada 38 seg, dímero D 2,5
mg/L, hematocrito 22%, recuento de leucocitos 2.300/uL, recuento de
plaquetas 22.000/uL, tiempo de protrombina 25 seg, albúmina 3,5 g/dL, alanina
aminotransferasa 30 U /L, aspartato aminotransferasa 35 U/L, bilirrubina total
0,9 mg/dL, fibrinógeno 60 mg/dL. ¿Cuál de los siguientes es el diagnóstico más
probable?
a. Anemia aplásica
b. Coagulopatía de la enfermedad hepática.
c. Coagulación intravascular diseminada
d. Púrpura trombocitopénica trombótica
e. Síndrome de Wiskott Aldrich
Evaluación y manejo de la coagulación intravascular diseminada (CID) en
adultos (uptodate)
• A un hombre de 65 años se le diagnostica una embolia pulmonar
segmentaria múltiple en el pulmón derecho. No tiene otros problemas
médicos y no toma medicamentos. En el examen físico, la presión
arterial es de 132/76 mm Hg, la frecuencia del pulso es de 100/min y la
frecuencia respiratoria es de 18/min. La saturación de oxígeno es del
98% en reposo respirando aire ambiente. El paciente está alerta y
comprende rápidamente las implicancias diagnósticas y la terapia
requerida. No requiere analgésicos. El paciente participa en un proceso
compartido de toma de decisiones con respecto al tratamiento. ¿Cuál de
las siguientes es la recomendación más adecuada?
a. Alta a domicilio y tratamiento con dabigatrán
b. Alta a domicilio y tratamiento con rivaroxabán
c. Hospitalizar y tratar con terapia trombolítica
d. Hospitalizar y tratar con heparina no fraccionada y warfarina
e. Alta a domicilio y tratamiento con clopidogrel
• A un hombre de 65 años se le diagnostica una embolia pulmonar
segmentaria múltiple en el pulmón derecho. No tiene otros problemas
médicos y no toma medicamentos. En el examen físico, la presión
arterial es de 132/76 mm Hg, la frecuencia del pulso es de 100/min y la
frecuencia respiratoria es de 18/min. La saturación de oxígeno es del
98% en reposo respirando aire ambiente. El paciente está alerta y
comprende rápidamente las implicancias diagnósticas y la terapia
requerida. No requiere analgésicos. El paciente participa en un proceso
compartido de toma de decisiones con respecto al tratamiento. ¿Cuál de
las siguientes es la recomendación más adecuada?
a. Alta a domicilio y tratamiento con dabigatrán
b. Alta a domicilio y tratamiento con rivaroxabán
c. Hospitalizar y tratar con terapia trombolítica
d. Hospitalizar y tratar con heparina no fraccionada y warfarina
e. Alta a domicilio y tratamiento con clopidogrel
Tratamiento, pronóstico y seguimiento de la embolia pulmonar aguda en
adultos (uptodate)
• Un hombre de 58 años es evaluado en urgencias por fiebre, dolor de cabeza. y rigidez
cervical. La fiebre empezó hace 2 días; la cefalea y la rigidez del cuello estaban presentes
al despertar hoy y se han vuelto severos. Hace dos semanas le diagnosticaron neuralgia
del trigémino y se inició tratamiento con carbamazepina. En el examen físico. la
temperatura es de 38,7 °C, la presión arterial es de 110/70 mm Hg, la frecuencia del pulso
es de 110/min y la frecuencia respiratoria es de 18/min. Hay rigidez de nuca, pero el resto
del examen neurológico es normal. No tiene papiledema. Se observan petequias y
equimosis en las extremidades inferiores. Estudios de laboratorio: tiempo de tromboplastina
parcial activada 36 seg, haptoglobina normal, hemoglobina 13,9 g/dl, recuento de
leucocitos 18.100/uL, recuento de plaquetas 30.000/uL, tiempo de protrombina 11,5 seg,
lactato deshidrogenasa normal. Un frotis de sangre periférica revela disminución de
plaquetas y morfología normal de eritrocitos. Una prueba de antiglobulina directa es
negativa. Se suspende la carbamazepina. Se obtienen hemocultivos. Ante una presunta
meningitis bacteriana se inician dexametasona y antibióticos empíricos. ¿Cuál de los
siguientes es el más apropiado en el manejo?
a. Punción lumbar
b. Intercambio de plasma
c. Transfusión de plaquetas
d. Plasma fresco congelado
e. No hay intervenciones adicionales
• Un hombre de 58 años es evaluado en urgencias por fiebre, dolor de cabeza. y rigidez
cervical. La fiebre empezó hace 2 días; la cefalea y la rigidez del cuello estaban presentes
al despertar hoy y se han vuelto severos. Hace dos semanas le diagnosticaron neuralgia
del trigémino y se inició tratamiento con carbamazepina. En el examen físico. la
temperatura es de 38,7 °C, la presión arterial es de 110/70 mm Hg, la frecuencia del pulso
es de 110/min y la frecuencia respiratoria es de 18/min. Hay rigidez de nuca, pero el resto
del examen neurológico es normal. No tiene papiledema. Se observan petequias y
equimosis en las extremidades inferiores. Estudios de laboratorio: tiempo de tromboplastina
parcial activada 36 seg, haptoglobina normal, hemoglobina 13,9 g/dl, recuento de
leucocitos 18.100/uL, recuento de plaquetas 30.000/uL, tiempo de protrombina 11,5 seg,
lactato deshidrogenasa normal. Un frotis de sangre periférica revela disminución de
plaquetas y morfología normal de eritrocitos. Una prueba de antiglobulina directa es
negativa. Se suspende la carbamazepina. Se obtienen hemocultivos. Ante una presunta
meningitis bacteriana se inician dexametasona y antibióticos empíricos. ¿Cuál de los
siguientes es el más apropiado en el manejo?
a. Punción lumbar
b. Intercambio de plasma
c. Transfusión de plaquetas
d. Plasma fresco congelado
e. No hay intervenciones adicionales
Transfusión de plaquetas: indicaciones, pedidos y riesgos asociados
(uptodate)
• Mujer de 28 años evaluada por fiebre y faringitis de 3 días.
Antecedentes médicos: hipertiroidismo diagnosticado hace 3 meses.
Consume una dieta vegetariana. Su única medicación es el metimazol.
Al examen físico la temperatura es de 38,2 °C; otros signos vitales son
normales. Se observa eritema faríngeo posterior. Estudios de
laboratorio: Hemoglobina 13,9 g/dL, recuento de leucocitos 2.300/uL
(20% neutrófilos, 72% linfocitos, 8% monocitos), recuento absoluto de
neutrófilos 460/uL, volumen corpuscular medio 92 fl, recuento de
plaquetas 302.000/uL. ¿Cuál de los siguientes es el diagnóstico más
probable?
a. Neutropenia étnica benigna
b. Neutropenia cíclica
c. Neutropenia inducida por fármacos
d. Deficiencia de vitamina B12
e. Deficiencia de folato
• Mujer de 28 años evaluada por fiebre y faringitis de 3 días.
Antecedentes médicos: hipertiroidismo diagnosticado hace 3 meses.
Consume una dieta vegetariana. Su única medicación es el metimazol.
Al examen físico la temperatura es de 38,2 °C; otros signos vitales son
normales. Se observa eritema faríngeo posterior. Estudios de
laboratorio: Hemoglobina 13,9 g/dL, recuento de leucocitos 2.300/uL
(20% neutrófilos, 72% linfocitos, 8% monocitos), recuento absoluto de
neutrófilos 460/uL, volumen corpuscular medio 92 fl, recuento de
plaquetas 302.000/uL. ¿Cuál de los siguientes es el diagnóstico más
probable?
a. Neutropenia étnica benigna
b. Neutropenia cíclica
c. Neutropenia inducida por fármacos
d. Deficiencia de vitamina B12
e. Deficiencia de folato
Abordaje del adulto con neutropenia inexplicable (uptodate)
• Un hombre de 70 años es hospitalizado con antecedentes de 3 días de dolor
abdominal agudo y náuseas y vómitos. Los antecedentes médicos y familiares
no son contributivos y no toma medicamentos. Al examen físico los signos
vitales son normales. La palpación provoca dolor leve a la palpación en la parte
superior del abdomen sin hepatoesplenomegalia ni masa abdominal. Los
estudios de laboratorio muestran un nivel de hemoglobina de 14,5 g/dl,
recuento de leucocitos de 5.000/uL y recuento de plaquetas de 250.000/uL. La
TC del abdomen con contraste demuestra una trombosis aguda de la vena
porta. ¿Cuál de las siguientes es la prueba diagnóstica adicional más
adecuada?
a. Medición de antitrombina
b. Mutación de tirosina quinasa JAK2
c. Medición de proteína C
d. Medición de proteína S
e. Dímero D
• Un hombre de 70 años es hospitalizado con antecedentes de 3 días de dolor
abdominal agudo y náuseas y vómitos. Los antecedentes médicos y familiares
no son contributivos y no toma medicamentos. Al examen físico los signos
vitales son normales. La palpación provoca dolor leve a la palpación en la parte
superior del abdomen sin hepatoesplenomegalia ni masa abdominal. Los
estudios de laboratorio muestran un nivel de hemoglobina de 14,5 g/dl,
recuento de leucocitos de 5.000/uL y recuento de plaquetas de 250.000/uL. La
TC del abdomen con contraste demuestra una trombosis aguda de la vena
porta. ¿Cuál de las siguientes es la prueba diagnóstica adicional más
adecuada?
a. Medición de antitrombina
b. Mutación de tirosina quinasa JAK2
c. Medición de proteína C
d. Medición de proteína S
e. Dímero D
Trombosis aguda de la vena porta en adultos: manifestaciones clínicas,
diagnóstico y tratamiento (uptodate)
Síndrome de Budd-Chiari: epidemiología, manifestaciones clínicas y
diagnóstico (uptodate)
• Un hombre de 74 años es evaluado en el hospital por trombocitopenia de nueva
aparición. Fue hospitalizado hace 48 horas por tratamiento de embolia pulmonar
confirmada por angiografía por tomografía computarizada. El historial médico es
importante para la cirugía de injerto de derivación de arteria coronaria hace 3 semanas.
Los medicamentos son aspirinas. clopidogrel, metoprolol. lisinopril, furosemida y
atorvastatina; Se inició heparina de bajo peso molecular al ingreso al hospital. En el
examen físico. los signos vitales son normales. Aparte de una cicatriz de esternotomía
que cicatriza bien, el examen físico no presenta nada especial. Estudios de laboratorio:
Hemoglobina 12g/dL, recuento de leucocitos 11.500/uL, recuento de plaquetas
90.000/uL (250.000 en el momento de la admisión). La tasa de filtración glomerular
estimada fue >60 ml/min/1,73 m2. El frotis de sangre periférica es normal. Se suspende
la heparina de bajo peso molecular y se solicita una prueba de anticuerpos inducidos
por heparina. ¿Cuál de los siguientes es el manejo más adecuado?
a. Suspender la aspirina y el clopidogrel
b. Insertar un filtro de vena cava inferior
c. Iniciar rivaroxabán o apixabán
d. Iniciar warfarina
e. Reiniciar la heparina en la medida de lo posible
• Un hombre de 74 años es evaluado en el hospital por trombocitopenia de nueva
aparición. Fue hospitalizado hace 48 horas por tratamiento de embolia pulmonar
confirmada por angiografía por tomografía computarizada. El historial médico es
importante para la cirugía de injerto de derivación de arteria coronaria hace 3 semanas.
Los medicamentos son aspirinas. clopidogrel, metoprolol. lisinopril, furosemida y
atorvastatina; Se inició heparina de bajo peso molecular al ingreso al hospital. En el
examen físico. los signos vitales son normales. Aparte de una cicatriz de esternotomía
que cicatriza bien, el examen físico no presenta nada especial. Estudios de laboratorio:
Hemoglobina 12g/dL, recuento de leucocitos 11.500/uL, recuento de plaquetas
90.000/uL (250.000 en el momento de la admisión). La tasa de filtración glomerular
estimada fue >60 ml/min/1,73 m2. El frotis de sangre periférica es normal. Se suspende
la heparina de bajo peso molecular y se solicita una prueba de anticuerpos inducidos
por heparina. ¿Cuál de los siguientes es el manejo más adecuado?
a. Suspender la aspirina y el clopidogrel
b. Insertar un filtro de vena cava inferior
c. Iniciar rivaroxabán o apixabán
d. Iniciar warfarina
e. Reiniciar la heparina en la medida de lo posible
Manejo de la trombocitopenia inducida por heparina (uptodate)
• Un hombre de 63 años es evaluado por aparición de hematomas con facilidad,
empeoramiento del edema y mareos al ponerse de pie durante los últimos 9
meses. Por lo demás, el historial médico no contribuye; no toma
medicamentos. En el examen físico, la presión arterial es de 105/60 mm Hg en
posición sentada y de 80/50 mm Hg en posición de pie; La frecuencia del pulso
es de 105/min tanto sentado como de pie. Hay equimosis periorbitarias,
macroglosia y distensión venosa yugular. Los estudios de laboratorio muestran
un nivel de hemoglobina normal, un nivel de albúmina sérica de 2 g/dl y un
nivel de creatinina sérica de 1,5 mg/dl. La excreción de albúmina en orina de
24 horas es de 3500 mg. La electroforesis de proteínas séricas muestra un
pico lambda de IgG de 1,2 mg/dl. El ecocardiograma muestra hipertrofia
miocárdica biventricular con ventrículos no dilatados y disfunción diastólica. La
fracción de eyección del ventrículo izquierdo es del 51%. ¿Cuál de las
siguientes es la prueba diagnóstica más adecuada a realizar a continuación?
a. Biopsia de grasa abdominal periumbilical
b. Biopsia endomiocárdica por tomografía
c. Biopsia de riñón guiada por ecografía
d. Prueba de mesa inclinable (tilt test)
e. Péptidos natriuréticos (ProBNP)
• Un hombre de 63 años es evaluado por aparición de hematomas con facilidad,
empeoramiento del edema y mareos al ponerse de pie durante los últimos 9
meses. Por lo demás, el historial médico no contribuye; no toma
medicamentos. En el examen físico, la presión arterial es de 105/60 mm Hg en
posición sentada y de 80/50 mm Hg en posición de pie; La frecuencia del pulso
es de 105/min tanto sentado como de pie. Hay equimosis periorbitarias,
macroglosia y distensión venosa yugular. Los estudios de laboratorio muestran
un nivel de hemoglobina normal, un nivel de albúmina sérica de 2 g/dl y un
nivel de creatinina sérica de 1,5 mg/dl. La excreción de albúmina en orina de
24 horas es de 3500 mg. La electroforesis de proteínas séricas muestra un
pico lambda de IgG de 1,2 mg/dl. El ecocardiograma muestra hipertrofia
miocárdica biventricular con ventrículos no dilatados y disfunción diastólica. La
fracción de eyección del ventrículo izquierdo es del 51%. ¿Cuál de las
siguientes es la prueba diagnóstica más adecuada a realizar a continuación?
a. Biopsia de grasa abdominal periumbilical
b. Biopsia endomiocárdica por tomografía
c. Biopsia de riñón guiada por ecografía
d. Prueba de mesa inclinable (tilt test)
e. Péptidos natriuréticos (ProBNP)
Macroglobulinemia de Waldenström. Enfermedades de cadenas pesadas.
Crioglobulinemia. Amiloidosis.
Hematología de Pregrado 4ta edición
• Un hombre de 30 años es evaluado por fatiga progresiva de varios meses de
duración. Por lo demás, el historial médico no tiene nada de especial. Indica
tener un hermano. No toma medicamentos. En el examen físico, la frecuencia
del pulso es de 114/min; otros signos vitales son normales. La saturación de
oxígeno es del 98%, respirando aire ambiente. Se observa palidez del borde
conjuntival y petequias dispersas. estudios tabatorios: Hemoglobina 7,1 g/dL,
recuento de leucocitos 1.200/uL (65% neutrófilos, 35% linfocitos), volumen
corpuscular medio 90 fl, recuento de plaquetas 22.000/uL, recuento de
reticulocitos 1% de eritrocitos. El frotis de sangre periférica muestra eritrocitos
con morfología normal y plaquetas disminuidas. Una biopsia de médula ósea
muestra una marcada hipocelularidad y un mayor contenido de grasa sin
células displásicas. La prueba de hepatitis viral es negativa. Los resultados de
las pruebas del hermano muestran una coincidencia HLA. ¿Cuál de las
siguientes es la terapia más eficaz?
a. Trasplante alogénico de células madre hematopoyéticas
b. Globulina antitimocítica
c. Azacitidina
d. Inmunoglobulina intravenosa
e. Corticosteroides
• Un hombre de 30 años es evaluado por fatiga progresiva de varios meses de
duración. Por lo demás, el historial médico no tiene nada de especial. Indica
tener un hermano. No toma medicamentos. En el examen físico, la frecuencia
del pulso es de 114/min; otros signos vitales son normales. La saturación de
oxígeno es del 98%, respirando aire ambiente. Se observa palidez del borde
conjuntival y petequias dispersas. estudios tabatorios: Hemoglobina 7,1 g/dL,
recuento de leucocitos 1.200/uL (65% neutrófilos, 35% linfocitos), volumen
corpuscular medio 90 fl, recuento de plaquetas 22.000/uL, recuento de
reticulocitos 1% de eritrocitos. El frotis de sangre periférica muestra eritrocitos
con morfología normal y plaquetas disminuidas. Una biopsia de médula ósea
muestra una marcada hipocelularidad y un mayor contenido de grasa sin
células displásicas. La prueba de hepatitis viral es negativa. Los resultados de
las pruebas del hermano muestran una coincidencia HLA. ¿Cuál de las
siguientes es la terapia más eficaz?
a. Trasplante alogénico de células madre hematopoyéticas
b. Globulina antitimocítica
c. Azacitidina
d. Inmunoglobulina intravenosa
e. Corticosteroides
Tratamiento de la anemia aplásica en adultos (uptodate)
• Un hombre de 75 años es evaluado por un INR elevado encontrado en
un monitoreo de rutina. Por lo demás se encuentra bien y no tiene
evidencia de sangrado. El historial médico es importante para la
flbrilación auricular. Su única medicación es la warfarina. Los estudios
de laboratorio muestran un nivel de hemoglobina normal y un INR de
8,5. Se indica al paciente que no tome warfarina hasta una evaluación
adicional del INR. ¿Cuál de las siguientes es la gestión adicional más
adecuada?
a. Concentrado de complejo de protrombina de 4 factores
b. Factor VIIa
c. Plasma fresco congelado
d. Vitamina K
e. Observación
• Un hombre de 75 años es evaluado por un INR elevado encontrado en
un monitoreo de rutina. Por lo demás se encuentra bien y no tiene
evidencia de sangrado. El historial médico es importante para la
flbrilación auricular. Su única medicación es la warfarina. Los estudios
de laboratorio muestran un nivel de hemoglobina normal y un INR de
8,5. Se indica al paciente que no tome warfarina hasta una evaluación
adicional del INR. ¿Cuál de las siguientes es la gestión adicional más
adecuada?
a. Concentrado de complejo de protrombina de 4 factores
b. Factor VIIa
c. Plasma fresco congelado
d. Vitamina K
e. Observación
Manejo del sangrado asociado a warfarina o INR supraterapéutico (uptodate)
• Un hombre de 77 años es evaluado por anemia que ha empeorado durante el
último año. Presenta fatiga y disnea de esfuerzo. Tiene antecedentes de
hipertensión arterial mal controlada. Los medicamentos son lisinopril,
amlodipino e hidroclorotiazida. En el examen físico la presión arterial es de
180/100 mm Hg. El resto de los signos vitales y la exploración física son
normales. Estudios de laboratorio: Hemoglobina 8,8 g/dL, recuento de
leucocitos 6.800/uL, volumen corpuscular medio 88 fL, recuento de plaquetas
414.000/uL, recuento de reticulocitos 1,2% de los eritrocitos, creatinina 3,2
mg/dL, ferritina 150 ng/ mL, hierro 150 ng/mL, capacidad total de fijación de
hierro 300 pg/dl. Las heces son negativas para sangre oculta. El frotis de
sangre periférica demuestra una morfología normal de leucocitos y eritrocitos.
La ecografía muestra pequeños riñones ecogénicos bilateralmente. ¿Cuál de
los siguientes es el diagnóstico más probable?
a. Anemia de inflamación
b. Anemia de enfermedad renal
c. Anemia hemolítica microangiopática
d. Síndrome mielodisplásico
e. Anemia sideroblástica
• Un hombre de 77 años es evaluado por anemia que ha empeorado durante el
último año. Presenta fatiga y disnea de esfuerzo. Tiene antecedentes de
hipertensión arterial mal controlada. Los medicamentos son lisinopril,
amlodipino e hidroclorotiazida. En el examen físico la presión arterial es de
180/100 mm Hg. El resto de los signos vitales y la exploración física son
normales. Estudios de laboratorio: Hemoglobina 8,8 g/dL, recuento de
leucocitos 6.800/uL, volumen corpuscular medio 88 fL, recuento de plaquetas
414.000/uL, recuento de reticulocitos 1,2% de los eritrocitos, creatinina 3,2
mg/dL, ferritina 150 ng/ mL, hierro 150 ng/mL, capacidad total de fijación de
hierro 300 pg/dl. Las heces son negativas para sangre oculta. El frotis de
sangre periférica demuestra una morfología normal de leucocitos y eritrocitos.
La ecografía muestra pequeños riñones ecogénicos bilateralmente. ¿Cuál de
los siguientes es el diagnóstico más probable?
a. Anemia de inflamación
b. Anemia de enfermedad renal
c. Anemia hemolítica microangiopática
d. Síndrome mielodisplásico
e. Anemia sideroblástica
Descripción general del tratamiento de la enfermedad renal crónica en adultos
(uptodate)
• Una mujer de 68 años acude a consulta 3 semanas después de una
hemicolectomía derecha por cáncer de colon en estadio III. Por lo
demás, el historial médico no es destacable y no toma medicamentos. El
examen físico no presenta complicaciones y el sitio quirúrgico está
cicatrizando bien. Está prevista quimioterapia con 5 fluorouracilo,
leucovorina y oxaliplatino. ¿Cuál de los siguientes es el manejo del
riesgo de tromboembolismo venoso más apropiado para este paciente?
a. Calcular la puntuación APACHE
b. Calcular la puntuación de Khorana
c. Iniciar apixabán
d. Iniciar heparina de bajo peso molecular
e. Iniciar warfarina
• Una mujer de 68 años acude a consulta 3 semanas después de una
hemicolectomía derecha por cáncer de colon en estadio III. Por lo
demás, el historial médico no es destacable y no toma medicamentos. El
examen físico no presenta complicaciones y el sitio quirúrgico está
cicatrizando bien. Está prevista quimioterapia con 5 fluorouracilo,
leucovorina y oxaliplatino. ¿Cuál de los siguientes es el manejo del
riesgo de tromboembolismo venoso más apropiado para este paciente?
a. Calcular la puntuación APACHE
b. Calcular la puntuación de Khorana
c. Iniciar apixabán
d. Iniciar heparina de bajo peso molecular
e. Iniciar warfarina
Riesgo y prevención del tromboembolismo venoso en adultos con cáncer
(uptodate)
• Un hombre de 35 años es evaluado durante una visita de rutina por beta-
talasemia intermedia. Hace un año comenzó a necesitar transfusiones de
eritrocitos cada 4 a 6 meses por anemia sintomática. No presenta otros
síntomas ni problemas médicos y su única medicación es el ácido fólico. Al
examen físico los signos vitales son normales. La prominencia frontal, la
macrocefalia, la ictericia escleral y la esplenomegalia están presentes y no han
cambiado con respecto al examen anterior. El resto del examen físico es
normal. Los estudios de laboratorio muestran un nivel de hemoglobina de 7,9
g/dl, leucocitos de 6.400/uL y plaquetas de 110.000/uL. Los estudios de hierro
estaban en el límite superior de lo normal hace un año y medio. ¿Cuál de las
siguientes es la opción de manejo adicional más apropiada?
a. Ultrasonografía abdominal
b. Estudios de hierro
c. Anticoagulación profiláctica
d. Esplenectomía
e. Biopsia hepática
• Un hombre de 35 años es evaluado durante una visita de rutina por beta-
talasemia intermedia. Hace un año comenzó a necesitar transfusiones de
eritrocitos cada 4 a 6 meses por anemia sintomática. No presenta otros
síntomas ni problemas médicos y su única medicación es el ácido fólico. Al
examen físico los signos vitales son normales. La prominencia frontal, la
macrocefalia, la ictericia escleral y la esplenomegalia están presentes y no han
cambiado con respecto al examen anterior. El resto del examen físico es
normal. Los estudios de laboratorio muestran un nivel de hemoglobina de 7,9
g/dl, leucocitos de 6.400/uL y plaquetas de 110.000/uL. Los estudios de hierro
estaban en el límite superior de lo normal hace un año y medio. ¿Cuál de las
siguientes es la opción de manejo adicional más apropiada?
a. Ultrasonografía abdominal
b. Estudios de hierro
c. Anticoagulación profiláctica
d. Esplenectomía
e. Biopsia hepática
Abordaje del paciente con sospecha de sobrecarga de hierro (uptodate)
Abordaje del paciente con sospecha de sobrecarga de hierro (uptodate)
• Un hombre de 32 años es evaluado por hematomas fáciles. No tiene otros
problemas médicos; no toma medicamentos, drogas recreativas ni
suplementos y no bebe alcohol. Al examen físico los signos vitales son
normales. Se observan dos pequeñas equimosis en el muslo izquierdo;
También tiene un pequeño hematoma en la parte superior del brazo derecho.
No se ven petequias. No presenta linfadenopatías ni hepatoesplenomegalia.
Estudios de laboratorio: Hemoglobina 14,5 g/dL, recuento de leucocitos
5500/uL, volumen corpuscular medio 85 fL, recuento de plaquetas 44.000/uL,
recuento de reticulocitos 1,2% de eritrocitos, creatinina 0,7 mg/dl. Urianálisis
normal. El frotis de sangre periférica es normal. ¿Cuáles de las siguientes son
las pruebas diagnósticas más adecuadas?
a. Anticuerpos antinucleares y nivel de complemento.
b. Anticuerpos antiplaquetarios y prueba directa de antiglobulina
c. Virus VIH y hepatitis C
d. Niveles de vitamina B y folato.
e. Biopsia de médula ósea
• Un hombre de 32 años es evaluado por hematomas fáciles. No tiene otros
problemas médicos; no toma medicamentos, drogas recreativas ni
suplementos y no bebe alcohol. Al examen físico los signos vitales son
normales. Se observan dos pequeñas equimosis en el muslo izquierdo;
También tiene un pequeño hematoma en la parte superior del brazo derecho.
No se ven petequias. No presenta linfadenopatías ni hepatoesplenomegalia.
Estudios de laboratorio: Hemoglobina 14,5 g/dL, recuento de leucocitos
5500/uL, volumen corpuscular medio 85 fL, recuento de plaquetas 44.000/uL,
recuento de reticulocitos 1,2% de eritrocitos, creatinina 0,7 mg/dl. Urianálisis
normal. El frotis de sangre periférica es normal. ¿Cuáles de las siguientes son
las pruebas diagnósticas más adecuadas?
a. Anticuerpos antinucleares y nivel de complemento.
b. Anticuerpos antiplaquetarios y prueba directa de antiglobulina
c. Virus VIH y hepatitis C
d. Niveles de vitamina B y folato.
e. Biopsia de médula ósea
Trombocitopenia inmune (PTI) en adultos: manifestaciones clínicas y
diagnóstico (uptodate)
Un hombre de 72 años es evaluado por fatiga progresiva de varios meses de
duración. Como antecedente destaca carcinoma de vejiga diagnosticado hace 6
años, el cual fue tratado con combinación de quimioterapia y radiación, cistectomía
y creación de bolsa de derivación urinaria con resección de 50 cm de íleon. Toma
omeprazol para la enfermedad por reflujo gastroesofágico. Los signos vitales y el
resto del examen físico no contribuyen. Estudios de laboratorio: Hemoglobina 8,8
g/dL, recuento de leucocitos 4000/uL, volumen corpuscular medio 110 fL, recuento
de plaquetas 140.000/uL, recuento de reticulocitos 1% de los eritrocitos. Alanina
aminotransferasa 30 U/L, aspartato aminotransferasa 50 U/L, bilirrubina total 3,5
mg/dL, bilirrubina directa 0,5 mg/dL, lactato deshidrogenasa 400 U/L. Se muestra un
frotis de sangre periférica. ¿Cuál de las siguientes es la causa más probable de la
anemia de este paciente?

a. Hipotiroidismo
b. Anemia hemolítica inmunomediada
c. Absorción alterada de vitamina B12
d. Enfermedad del hígado
e. Falla renal crónica
Un hombre de 72 años es evaluado por fatiga progresiva de varios meses de
duración. Como antecedente destaca carcinoma de vejiga diagnosticado hace 6
años, el cual fue tratado con combinación de quimioterapia y radiación, cistectomía
y creación de bolsa de derivación urinaria con resección de 50 cm de íleon. Toma
omeprazol para la enfermedad por reflujo gastroesofágico. Los signos vitales y el
resto del examen físico no contribuyen. Estudios de laboratorio: Hemoglobina 8,8
g/dL, recuento de leucocitos 4000/uL, volumen corpuscular medio 110 fL, recuento
de plaquetas 140.000/uL, recuento de reticulocitos 1% de los eritrocitos. Alanina
aminotransferasa 30 U/L, aspartato aminotransferasa 50 U/L, bilirrubina total 3,5
mg/dL, bilirrubina directa 0,5 mg/dL, lactato deshidrogenasa 400 U/L. Se muestra un
frotis de sangre periférica. ¿Cuál de las siguientes es la causa más probable de la
anemia de este paciente?

a. Hipotiroidismo
b. Anemia hemolítica inmunomediada
c. Absorción alterada de vitamina B12
d. Enfermedad del hígado
e. Falla renal crónica
Anemia megaloblástica
Pregrado de Hematología (4ta ed)
• Una mujer de 48 años es evaluada en el servicio de urgencias por hinchazón y
dolor en la pierna derecha de 1 semana de duración. No informa ninguna
cirugía reciente, lesiones, períodos de inmovilidad o vuelos largos en avión. El
historial médico no tiene nada de especial. Ella no toma medicamentos. Al
examen físico los signos vitales son normales. El IMC es 20. La saturación de
oxígeno es del 99% respirando aire ambiente. El edema con fóvea se extiende
hasta la rodilla de la pierna derecha y la pantorrilla derecha es 4 cm más
grande que la izquierda. Los pulsos están intactos y el llenado capilar es
normal. La ecografía dúplex muestra una trombosis aguda de la vena femoral
derecha. ¿Cuál de los siguientes es el manejo más adecuado?
a. Anticoagulación durante 3 meses.
b. Anticoagulación durante 3 meses más filtro de vena cava inferior.
c. Anticoagulación prolongada
d. Terapia trombolítica
e. Aspirina más clopidogrel
• Una mujer de 48 años es evaluada en el servicio de urgencias por hinchazón y
dolor en la pierna derecha de 1 semana de duración. No informa ninguna
cirugía reciente, lesiones, períodos de inmovilidad o vuelos largos en avión. El
historial médico no tiene nada de especial. Ella no toma medicamentos. Al
examen físico los signos vitales son normales. El IMC es 20. La saturación de
oxígeno es del 99% respirando aire ambiente. El edema con fóvea se extiende
hasta la rodilla de la pierna derecha y la pantorrilla derecha es 4 cm más
grande que la izquierda. Los pulsos están intactos y el llenado capilar es
normal. La ecografía dúplex muestra una trombosis aguda de la vena femoral
derecha. ¿Cuál de los siguientes es el manejo más adecuado?
a. Anticoagulación durante 3 meses.
b. Anticoagulación durante 3 meses más filtro de vena cava inferior.
c. Anticoagulación prolongada
d. Terapia trombolítica
e. Aspirina más clopidogrel
Descripción general del tratamiento de la trombosis venosa profunda (TVP) de
las extremidades inferiores proximal y distal (uptodate)
• Un hombre de 70 años es evaluado por eritrocitosis descubierta recientemente.
Aparte de la somnolencia diurna, se siente bien. Tiene un historial de
tabaquismo de 50 paquetes al año y continúa fumando. También padece
obesidad, hipertensión y apnea obstructiva del sueño. No cumple con la
ventilación con presión positiva continua en las vías respiratorias (CPAP)
debido a malestar. Su única medicación es lisinopril. Al examen físico los
signos vitales son normales. El IMC es 35. La saturación de oxígeno es del
98% respirando aire ambiente. El examen no tiene nada de especial. Estudios
de laboratorio: Eritropoyetina 33 mU/mL, hematocrito 52%, leucocitos 8.000/uL,
plaquetas 185.000/uL. Las pruebas genéticas son negativas para la mutación
JAK2V617F. Recibe una breve intervención conductual para dejar de fumar y
se le ofrece vareniclina. Se le refiere a la unidad de medicina del sueño para
discutir estrategias para disminuir las molestias con la CPAP y mejorar la
adherencia. ¿Cuál de las siguientes es la gestión adicional más adecuada?
a. Biopsia de médula ósea
b. Hidroxiurea
c. Flebotomía
d. Ruxolitinib
e. No hay más pruebas ni intervenciones
• Un hombre de 70 años es evaluado por eritrocitosis descubierta recientemente.
Aparte de la somnolencia diurna, se siente bien. Tiene un historial de
tabaquismo de 50 paquetes al año y continúa fumando. También padece
obesidad, hipertensión y apnea obstructiva del sueño. No cumple con la
ventilación con presión positiva continua en las vías respiratorias (CPAP)
debido a malestar. Su única medicación es lisinopril. Al examen físico los
signos vitales son normales. El IMC es 35. La saturación de oxígeno es del
98% respirando aire ambiente. El examen no tiene nada de especial. Estudios
de laboratorio: Eritropoyetina 33 mU/mL, hematocrito 52%, leucocitos 8.000/uL,
plaquetas 185.000/uL. Las pruebas genéticas son negativas para la mutación
JAK2V617F. Recibe una breve intervención conductual para dejar de fumar y
se le ofrece vareniclina. Se le refiere a la unidad de medicina del sueño para
discutir estrategias para disminuir las molestias con la CPAP y mejorar la
adherencia. ¿Cuál de las siguientes es la gestión adicional más adecuada?
a. Biopsia de médula ósea
b. Hidroxiurea
c. Flebotomía
d. Ruxolitinib
e. No hay más pruebas ni intervenciones
Policitemia vera y policitemia secundaria: tratamiento y pronóstico (uptodate)
Policitemia vera y policitemia secundaria: tratamiento y pronóstico (uptodate)
• Mujer de 27 años es evaluada por hematomas espontáneos. Ella informa
antecedentes de menstruaciones abundantes desde la menarquia, lo que
requirió tratamiento para la anemia por deficiencia de hierro. Su madre y su tío
materno también tienen antecedentes de hemorragia. Ella no toma
medicamentos. Al examen físico los signos vitales son normales. Presenta
equimosis en las extremidades, pero la textura de la piel y la movilidad articular
son normales. El resto del examen no tiene nada de especial. Estudios de
laboratorio: tiempo de tromboplastina parcial activada 37 seg, hemoglobina
11,5 g/dL, recuento de leucocitos 8.000/uL, volumen corpuscular medio 78 fL,
recuento de plaquetas 140.000/uL, tiempo de protrombina 13 seg, factor VIII
40%. El resultado de la prueba de función plaquetaria es anormal. ¿Cuál de los
siguientes es el diagnóstico más probable?
a. Síndrome de Ehlers-Danlos
b. Portador de hemofilia A
c. Púrpura trombocitopénica inmune
d. Púrpura trombocitopénica trombótica
e. enfermedad de von Willebrand
• Mujer de 27 años es evaluada por hematomas espontáneos. Ella informa
antecedentes de menstruaciones abundantes desde la menarquia, lo que
requirió tratamiento para la anemia por deficiencia de hierro. Su madre y su tío
materno también tienen antecedentes de hemorragia. Ella no toma
medicamentos. Al examen físico los signos vitales son normales. Presenta
equimosis en las extremidades, pero la textura de la piel y la movilidad articular
son normales. El resto del examen no tiene nada de especial. Estudios de
laboratorio: tiempo de tromboplastina parcial activada 37 seg, hemoglobina
11,5 g/dL, recuento de leucocitos 8.000/uL, volumen corpuscular medio 78 fL,
recuento de plaquetas 140.000/uL, tiempo de protrombina 13 seg, factor VIII
40%. El resultado de la prueba de función plaquetaria es anormal. ¿Cuál de los
siguientes es el diagnóstico más probable?
a. Síndrome de Ehlers-Danlos
b. Portador de hemofilia A
c. Púrpura trombocitopénica inmune
d. Púrpura trombocitopénica trombótica
e. enfermedad de von Willebrand
Presentación clínica y diagnóstico de la enfermedad de von Willebrand
(uptodate)
Presentación clínica y diagnóstico de la enfermedad de von Willebrand
(uptodate)
Un hombre de 75 años es hospitalizado con sepsis secundaria a neumonía adquirida en la
comunidad. Por lo demás, ya estaba mucho antes de su ingreso al hospital y no tomaba
medicamentos. En el examen físico, la temperatura es de 39,4 °C, la presión arterial es de
90/52 mm Hg, la frecuencia del pulso es 110/min y la frecuencia respiratoria es de 16/min.
La saturación de oxígeno es del 92% usando una máscara sin reinhalación. El examen
cardíaco revela taquicardia. Se observan crepitantes pulmonares basilares bilaterales.
Estudios de laboratorio: Tiempo de tromboplastina parcial activada 55 seg, dímero D 2,8
mg/L, hemoglobina 9,1 g/dL, recuento de leucocitos 14.000/uL, recuento de plaquetas
55.000/uL, tiempo de protrombina 19 seg, fibrinógeno 105 mg/dL. Se muestra un frotis de
sangre periférica. Se recogen cultivos. Se proporciona reanimación con líquidos con
solución salina 0,9'2 y se inician antibióticos de amplio espectro. ¿Cuál es el tratamiento
inicial más adecuado para la coagulopatía?

a. Eculizumab
b. prednisona
c. Intercambio de plasma terapéutico
d. Rivaroxabán
e. Sin tratamiento adicional
Un hombre de 75 años es hospitalizado con sepsis secundaria a neumonía adquirida en la
comunidad. Por lo demás, ya estaba mucho antes de su ingreso al hospital y no tomaba
medicamentos. En el examen físico, la temperatura es de 39,4 °C, la presión arterial es de
90/52 mm Hg, la frecuencia del pulso es 110/min y la frecuencia respiratoria es de 16/min.
La saturación de oxígeno es del 92% usando una máscara sin reinhalación. El examen
cardíaco revela taquicardia. Se observan crepitantes pulmonares basilares bilaterales.
Estudios de laboratorio: Tiempo de tromboplastina parcial activada 55 seg, dímero D 2,8
mg/L, hemoglobina 9,1 g/dL, recuento de leucocitos 14.000/uL, recuento de plaquetas
55.000/uL, tiempo de protrombina 19 seg, fibrinógeno 105 mg/dL. Se muestra un frotis de
sangre periférica. Se recogen cultivos. Se proporciona reanimación con líquidos con
solución salina 0,9'2 y se inician antibióticos de amplio espectro. ¿Cuál es el tratamiento
inicial más adecuado para la coagulopatía?

a. Eculizumab
b. Prednisona
c. Intercambio de plasma terapéutico
d. Rivaroxabán
e. Sin tratamiento adicional
Evaluación y manejo de la coagulación intravascular diseminada (CID) en
adultos (uptodate)
• Un hombre de 78 años está hospitalizado con disnea, aturdimiento,
palpitaciones y dolor torácico intermitente que progresa durante los últimos 7
días. El historial médico se destaca por enfermedad cardiovascular
aterosclerótica, hipertensión e hiperlipidemia. Los medicamentos son lisinopril,
atorvastatina, metoprolol y aspirina. Al examen físico la presión arterial es de
91/53 mmHg y el pulso de 107/min, la saturación de oxígeno es del 93%
respirando aire ambiente. Las conjuntivas están pálidas. El resto del examen
es normal. Estudios de laboratorio: Haptoglobina <10 mg/dL, hemoglobina 6,7
g/dL, recuento de leucocitos 9500/uL, recuento de plaquetas 315.000/uL,
recuento de reticulocitos 11% de los eritrocitos. Una prueba directa de
antiglobulina es fuertemente positiva para anti IgG y débilmente reactiva para
anticomplemento. El banco de sangre informa que todas las unidades
disponibles son incompatibles. ¿Cuál de los siguientes es el tratamiento
inmediato más adecuado?
a. Combinación de prednisona más rituximab
b. Transfusión de eritrocitos
c. Esplenectomía emergente
d. Metilprednisolona
e. Rituximab
• Un hombre de 78 años está hospitalizado con disnea, aturdimiento,
palpitaciones y dolor torácico intermitente que progresa durante los últimos 7
días. El historial médico se destaca por enfermedad cardiovascular
aterosclerótica, hipertensión e hiperlipidemia. Los medicamentos son lisinopril,
atorvastatina, metoprolol y aspirina. Al examen físico la presión arterial es de
91/53 mmHg y el pulso de 107/min, la saturación de oxígeno es del 93%
respirando aire ambiente. Las conjuntivas están pálidas. El resto del examen
es normal. Estudios de laboratorio: Haptoglobina <10 mg/dL, hemoglobina 6,7
g/dL, recuento de leucocitos 9500/uL, recuento de plaquetas 315.000/uL,
recuento de reticulocitos 11% de los eritrocitos. Una prueba directa de
antiglobulina es fuertemente positiva para anti IgG y débilmente reactiva para
anticomplemento. El banco de sangre informa que todas las unidades
disponibles son incompatibles. ¿Cuál de los siguientes es el tratamiento
inmediato más adecuado?
a. Combinación de prednisona más rituximab
b. Transfusión de eritrocitos
c. Esplenectomía emergente
d. Metilprednisolona
e. Rituximab
Transfusión de glóbulos rojos (RBC) en individuos con complejidad serológica
(uptodate)
• Una mujer de 59 años es evaluada por fatiga, sudores nocturnos intensos,
fiebre baja y pérdida de peso involuntaria durante los últimos 6 meses.
También describe dolor de cabeza, visión borrosa. visión doble y dificultad para
concentrarse. Los antecedentes médicos no son contributivos y no toma
medicamentos. Al examen físico los signos vitales son normales. El examen
fundoscópico revela venas retinianas dilatadas, hemorragias en llamas. y
papiledema. Hay sangre con costras en las fosas nasales. Hay linfadenopatía
voluminosa generalizada y hepatoesplenomegalia. Se observan numerosas
petequias en las extremidades inferiores. La electroforesis de proteínas séricas
y la inmunofijación muestran un pico de proteína monoclonal IgM kappa de 6
g/dl. La relación de cadenas ligeras libres es superior a 100. La viscosidad del
suero está pendiente. ¿Cuál de las siguientes es la terapia inmediata más
adecuada?
a. Hidroxiurea
b. Inmunoglobulina intravenosa
c. Plasmaféresis
d. Ultrafiltración
e. Corticosteroides
• Una mujer de 59 años es evaluada por fatiga, sudores nocturnos intensos,
fiebre baja y pérdida de peso involuntaria durante los últimos 6 meses.
También describe dolor de cabeza, visión borrosa. visión doble y dificultad para
concentrarse. Los antecedentes médicos no son contributivos y no toma
medicamentos. Al examen físico los signos vitales son normales. El examen
fundoscópico revela venas retinianas dilatadas, hemorragias en llamas. y
papiledema. Hay sangre con costras en las fosas nasales. Hay linfadenopatía
voluminosa generalizada y hepatoesplenomegalia. Se observan numerosas
petequias en las extremidades inferiores. La electroforesis de proteínas séricas
y la inmunofijación muestran un pico de proteína monoclonal IgM kappa de 6
g/dl. La relación de cadenas ligeras libres es superior a 100. La viscosidad del
suero está pendiente. ¿Cuál de las siguientes es la terapia inmediata más
adecuada?
a. Hidroxiurea
b. Inmunoglobulina intravenosa
c. Plasmaféresis
d. Ultrafiltración
e. Corticosteroides
Tratamiento y pronóstico de la macroglobulinemia de Waldenström (uptodate)
• Una mujer de 65 años es hospitalizada con diagnóstico de embolia pulmonar
segmentaria múltiple en los pulmones derecho e izquierdo. Por lo demás, el
historial médico no tiene nada de especial. Su única medicación es el
reemplazo hormonal de estrógeno-medroxiprogesterona, que ha sido
descontinuado. Al examen físico la presión arterial es de 124/64 mm Hg, la
frecuencia del pulso es de 124/min y la frecuencia respiratoria es de 24/min. La
saturación de oxígeno es del 93% respirando aire ambiente. Por lo demás, los
resultados del examen no contribuyen. El hemograma completo y el perfil
metabólico son normales. La tasa de filtración glomerular estimada es superior
a 60 ml/min/1,73 m2. ¿Cuál de los siguientes es el tratamiento más adecuado?
a. Warfarina
b. Apixabán
c. Filtro placentario de la vena cava inferior
d. Terapia trombolítica
e. Heparina no fraccionada
• Una mujer de 65 años es hospitalizada con diagnóstico de embolia pulmonar
segmentaria múltiple en los pulmones derecho e izquierdo. Por lo demás, el
historial médico no tiene nada de especial. Su única medicación es el
reemplazo hormonal de estrógeno-medroxiprogesterona, que ha sido
descontinuado. Al examen físico la presión arterial es de 124/64 mm Hg, la
frecuencia del pulso es de 124/min y la frecuencia respiratoria es de 24/min. La
saturación de oxígeno es del 93% respirando aire ambiente. Por lo demás, los
resultados del examen no contribuyen. El hemograma completo y el perfil
metabólico son normales. La tasa de filtración glomerular estimada es superior
a 60 ml/min/1,73 m2. ¿Cuál de los siguientes es el tratamiento más adecuado?
a. Warfarina
b. Apixabán
c. Filtro placentario de la vena cava inferior
d. Terapia trombolítica
e. Heparina no fraccionada
Tratamiento, pronóstico y seguimiento de la embolia pulmonar aguda en
adultos (uptodate)
• Una mujer de 21 años es evaluada por fatiga y disnea de esfuerzo que
progresa en los últimos 4 días junto con orina oscura. Hace una semana,
completó un tratamiento de 5 días con azitromicina para la tos y la fiebre, que
se resolvió con la medicación. Al examen físico los signos vitales son
normales. Se nota ictericia conjuntival. Estudios de laboratorio: Haptoglobina
<10mg/dL, hemoglobina 10,7 g/dL, recuento de leucocitos 8500/uL, recuento
de plaquetas 225.000/uL, recuento de reticulocitos 9% de los eritrocitos. La
prueba directa de antiglobulina es negativa para anti IgG y fuertemente
reactivo al anticomplemento. ¿Cuál de los siguientes es el manejo más
adecuado?
a. Título de aglutininas frías
b. Eculizumab
c. Citometría de flujo sanguíneo periférico para CD55 y CD59
d. Prednisona
e. Repetir la prueba de Coombs
• Una mujer de 21 años es evaluada por fatiga y disnea de esfuerzo que
progresa en los últimos 4 días junto con orina oscura. Hace una semana,
completó un tratamiento de 5 días con azitromicina para la tos y la fiebre, que
se resolvió con la medicación. Al examen físico los signos vitales son
normales. Se nota ictericia conjuntival. Estudios de laboratorio: Haptoglobina
<10mg/dL, hemoglobina 10,7 g/dL, recuento de leucocitos 8500/uL, recuento
de plaquetas 225.000/uL, recuento de reticulocitos 9% de los eritrocitos. La
prueba directa de antiglobulina es negativa para anti IgG y fuertemente
reactivo al anticomplemento. ¿Cuál de los siguientes es el manejo más
adecuado?
a. Título de aglutininas frías
b. Eculizumab
c. Citometría de flujo sanguíneo periférico para CD55 y CD59
d. Prednisona
e. Repetir la prueba de Coombs
Enfermedad de aglutininas frías (uptodate)
• Una mujer de 38 años es evaluada una semana antes de la
colecistectomía electiva. Los antecedentes médicos son importantes
para la anemia de células falciformes por hemoglobina SS con
aproximadamente uno o dos episodios de dolor vasooclusivo al año, que
se tratan en el ámbito ambulatorio. Los medicamentos son ácido fólico y
oxicodona según sea necesario durante los episodios de dolor. Los
estudios de laboratorio muestran un nivel de hemoglobina de 7,9 g/dl.
¿Cuál de los siguientes es el manejo perioperatorio más adecuado?
a. Transfusión simple postoperatoria; objetivo de hemoglobina 10 g/dl
b. Hidroxiurea preoperatoria
c. Exanguinotransfusión preoperatoria; objetivo de hemoglobina S inferior al 30%
d. Transfusión simple preoperatoria; objetivo de hemoglobina 10 g/dl
e. Prednisona preoperatoria y posoperatoria.
• Una mujer de 38 años es evaluada una semana antes de la
colecistectomía electiva. Los antecedentes médicos son importantes
para la anemia de células falciformes por hemoglobina SS con
aproximadamente uno o dos episodios de dolor vasooclusivo al año, que
se tratan en el ámbito ambulatorio. Los medicamentos son ácido fólico y
oxicodona según sea necesario durante los episodios de dolor. Los
estudios de laboratorio muestran un nivel de hemoglobina de 7,9 g/dl.
¿Cuál de los siguientes es el manejo perioperatorio más adecuado?
a. Transfusión simple postoperatoria; objetivo de hemoglobina 10 g/dl
b. Hidroxiurea preoperatoria
c. Exanguinotransfusión preoperatoria; objetivo de hemoglobina S inferior al 30%
d. Transfusión simple preoperatoria; objetivo de hemoglobina 10 g/dl
e. Prednisona preoperatoria y posoperatoria.
Manejo perioperatorio de adultos con anemia de células falciformes o talasemia
(uptodate)
• Una mujer de 65 años es evaluada en el departamento de emergencias por un
historial de 2 días de hinchazón en el brazo izquierdo y dolor moderado.
Recientemente le diagnosticaron osteomielitis del primer metatarsiano del pie
derecho y hace 2 semanas se le colocó un catéter central de inserción
periférica (PICC) izquierdo. Están previstas seis semanas de terapia con
antibióticos intravenosos. No toma otros medicamentos. Al examen físico los
signos vitales son normales. Presenta edema en miembro superior izquierdo.
Los pulsos son normales en las extremidades superiores y no se observa
hinchazón facial. La úlcera del pie está limpia y hay tejido de granulación. La
ecografía dúplex se realiza en la extremidad superior izquierda y demuestra
una trombosis venosa profunda aguda en las venas braquial y axilar. El PICC
está funcionando, bien posicionado y no está infectado. ¿Cuál de los siguientes
es el manejo más adecuado?
a. Anticoagulación
b. Remoción de PICC
c. Remoción de PICC y anticoagulación.
d. Terapia trombolítica
e. Remoción del PICC y trombectomía
• Una mujer de 65 años es evaluada en el departamento de emergencias por un
historial de 2 días de hinchazón en el brazo izquierdo y dolor moderado.
Recientemente le diagnosticaron osteomielitis del primer metatarsiano del pie
derecho y hace 2 semanas se le colocó un catéter central de inserción
periférica (PICC) izquierdo. Están previstas seis semanas de terapia con
antibióticos intravenosos. No toma otros medicamentos. Al examen físico los
signos vitales son normales. Presenta edema en miembro superior izquierdo.
Los pulsos son normales en las extremidades superiores y no se observa
hinchazón facial. La úlcera del pie está limpia y hay tejido de granulación. La
ecografía dúplex se realiza en la extremidad superior izquierda y demuestra
una trombosis venosa profunda aguda en las venas braquial y axilar. El PICC
está funcionando, bien posicionado y no está infectado. ¿Cuál de los siguientes
es el manejo más adecuado?
a. Anticoagulación
b. Remoción de PICC
c. Remoción de PICC y anticoagulación.
d. Terapia trombolítica
e. Remoción del PICC y trombectomía
Trombosis venosa de las extremidades superiores relacionada con catéter en
adultos (uptodate)
Una mujer de 36 años es evaluada por fatiga de 3 meses de duración y
disnea de esfuerzo de 1 mes de duración. Tiene 1 mes de posparto. Ella
no toma medicamentos. Al examen físico los signos vitales son normales.
Se nota palidez. No se observa ictericia escleral. El resto del examen es
normal. Estudios de laboratorio: Hemoglobina 7,9 g/dL (11,1 g/dL en el
primer trimestre), recuento de leucocitos 6800/uL, volumen corpuscular
medio 70 fl (80 fl en el primer trimestre), recuento de plaquetas
414.000/uL, recuento de reticulocitos 1% de los eritrocitos, índice de
anisocitosis elevado. Se muestra el frotis de sangre periférica. ¿Cuál de los
siguientes es el manejo más adecuado?

a. Colonoscopia
b. Hierro oral cada dos días
c. Hierro oral tres veces al día.
d. Electroforesis de hemoglobina
e. Prueba directa de Coombs
Una mujer de 36 años es evaluada por fatiga de 3 meses de duración y
disnea de esfuerzo de 1 mes de duración. Tiene 1 mes de posparto. Ella
no toma medicamentos. Al examen físico los signos vitales son normales.
Se nota palidez. No se observa ictericia escleral. El resto del examen es
normal. Estudios de laboratorio: Hemoglobina 7,9 g/dL (11,1 g/dL en el
primer trimestre), recuento de leucocitos 6800/uL, volumen corpuscular
medio 70 fl (80 fl en el primer trimestre), recuento de plaquetas
414.000/uL, recuento de reticulocitos 1% de los eritrocitos, índice de
anisocitosis elevado. Se muestra el frotis de sangre periférica. ¿Cuál de los
siguientes es el manejo más adecuado?

a. Colonoscopia
b. Hierro oral cada dos días
c. Hierro oral tres veces al día.
d. Electroforesis de hemoglobina
e. Prueba directa de Coombs
Tratamiento de la anemia por deficiencia de hierro en adultos (uptodate)
• Una mujer de 55 años es evaluada en el servicio de urgencias por
hinchazón de la pierna derecha y se le diagnostica una trombosis
venosa profunda proximal. El historial médico es importante para la
enfermedad renal en etapa terminal por la que se encuentra en
hemodiálisis. ¿Cuál de los siguientes es el tratamiento anticoagulante
más adecuado?
o Apixabán
o Dabigatrán
o Edoxabán
o Rivaroxabán
o Observación
• Una mujer de 55 años es evaluada en el servicio de urgencias por
hinchazón de la pierna derecha y se le diagnostica una trombosis
venosa profunda proximal. El historial médico es importante para la
enfermedad renal en etapa terminal por la que se encuentra en
hemodiálisis. ¿Cuál de los siguientes es el tratamiento anticoagulante
más adecuado?
o Apixabán
o Dabigatrán
o Edoxabán
o Rivaroxabán
o Observación
Anticoagulantes orales directos (ACOD) y anticoagulantes parenterales de
acción directa: posología y efectos adversos (uptodate)
• Mujer de 68 años es evaluada en urgencias por cuadro de melena de 2 días de
evolución, con dificultad para respirar y aturdimiento que comenzó hoy. El
historial médico es importante para la fibrilación auricular. Su única medicación
es la warfarina. En el examen físico, la presión arterial es de 82/50 mm Hg y la
frecuencia del pulso es de 126/min. y la frecuencia respiratoria es de 20/min.
La saturación de oxígeno es del 97%, respirando aire ambiente. Se observa
hemorragia de sangre de color rojo brillante por el recto. Los estudios de
laboratorio muestran un nivel de hemoglobina de 6,2 g/dL y un INR de 8,2. La
reanimación se inicia con transfusiones intravenosas de solución salina al 0,9%
y eritrocitos. Se administra vitamina K intravenosa. ¿Cuál de los siguientes es
el tratamiento adicional más adecuado?
a. Andexanet alfa
b. Concentrado de complejo de protrombina de 4 factores
c. Idarucizumab
d. Sulfato de protamina
e. Plasma fresco congelado
• Mujer de 68 años es evaluada en urgencias por cuadro de melena de 2 días de
evolución, con dificultad para respirar y aturdimiento que comenzó hoy. El
historial médico es importante para la fibrilación auricular. Su única medicación
es la warfarina. En el examen físico, la presión arterial es de 82/50 mm Hg y la
frecuencia del pulso es de 126/min. y la frecuencia respiratoria es de 20/min.
La saturación de oxígeno es del 97%, respirando aire ambiente. Se observa
hemorragia de sangre de color rojo brillante por el recto. Los estudios de
laboratorio muestran un nivel de hemoglobina de 6,2 g/dL y un INR de 8,2. La
reanimación se inicia con transfusiones intravenosas de solución salina al 0,9%
y eritrocitos. Se administra vitamina K intravenosa. ¿Cuál de los siguientes es
el tratamiento adicional más adecuado?
a. Andexanet alfa
b. Concentrado de complejo de protrombina de 4 factores
c. Idarucizumab
d. Sulfato de protamina
e. Plasma fresco congelado
Recomendaciones de las guías para el tratamiento de hemorragias asociadas a
warfarina y/o INR alto (uptodate)
• Mujer de 44 años es evaluada por hemocromatosis. A su padre le
diagnosticaron recientemente hemocromatosis tras una evaluación de
cirrosis criptogénica. Ella no toma medicamentos. En el examen físico,
los signos vitales y los hallazgos de la exploración son normales. Los
estudios de hierro muestran un nivel de ferritina de 100 ng/ml, un nivel
de hierro de 98 ug/dl y una saturación de transferrina del 35%. La
mutación heterocigótica C282Y del gen HFE se descubre mediante
pruebas genéticas, sin la mutación H63D o S65C. ¿Cuál de los
siguientes es el manejo más adecuado?
a. Terapia de quelación de hierro
b. Flebotomía
c. Estudios seriados de hierro
d. Sin intervención
e. Eritropoyetina
• Mujer de 44 años es evaluada por hemocromatosis. A su padre le
diagnosticaron recientemente hemocromatosis tras una evaluación de
cirrosis criptogénica. Ella no toma medicamentos. En el examen físico,
los signos vitales y los hallazgos de la exploración son normales. Los
estudios de hierro muestran un nivel de ferritina de 100 ng/ml, un nivel
de hierro de 98 ug/dl y una saturación de transferrina del 35%. La
mutación heterocigótica C282Y del gen HFE se descubre mediante
pruebas genéticas, sin la mutación H63D o S65C. ¿Cuál de los
siguientes es el manejo más adecuado?
a. Terapia de quelación de hierro
b. Flebotomía
c. Estudios seriados de hierro
d. Sin intervención
e. Eritropoyetina
Manejo y pronóstico de la hemocromatosis hereditaria (uptodate)
• Una mujer de 75 años es evaluada por fatiga y aturdimiento de 72 horas de
duración. Fue hospitalizada hace 2 meses con una trombosis venosa profunda
de la vena femoral izquierda y fue dada de alta con apixaban. Hace 1 semana
terminó un tratamiento de tres días con trimetoprim sulfametoxazol para una
infección del tracto urinario no complicada. No refiere otros síntomas, dolor en
el costado, dolor en las articulaciones ni cambios en el color de las heces. Su
única medicación es apixaban. Al examen físico los signos vitales son
normales. La saturación de oxígeno es del 98% respirando aire ambiente. No
presenta equimosis ni hematomas. Las heces son negativas para sangre
oculta. Estudios de laboratorio: Hemoglobina 9 g/dL, recuento de leucocitos
8.000/uL, volumen corpuscular medio 90 fL, recuento de plaquetas 280.000/uL.
El hemograma completo de hace 2 meses fue normal. ¿Cuál de las siguientes
es la siguiente prueba más apropiada para evaluar la anemia de este
paciente?
a. Aspiración y biopsia de médula ósea.
b. Prueba de antiglobulina directa
c. Recuento de reticulocitos y frotis de sangre periférica.
d. Niveles séricos de hierro y ferritina.
e. Niveles de vitamina B12 y folato.
• Una mujer de 75 años es evaluada por fatiga y aturdimiento de 72 horas de
duración. Fue hospitalizada hace 2 meses con una trombosis venosa profunda
de la vena femoral izquierda y fue dada de alta con apixaban. Hace 1 semana
terminó un tratamiento de tres días con trimetoprim sulfametoxazol para una
infección del tracto urinario no complicada. No refiere otros síntomas, dolor en
el costado, dolor en las articulaciones ni cambios en el color de las heces. Su
única medicación es apixaban. Al examen físico los signos vitales son
normales. La saturación de oxígeno es del 98% respirando aire ambiente. No
presenta equimosis ni hematomas. Las heces son negativas para sangre
oculta. Estudios de laboratorio: Hemoglobina 9 g/dL, recuento de leucocitos
8.000/uL, volumen corpuscular medio 90 fL, recuento de plaquetas 280.000/uL.
El hemograma completo de hace 2 meses fue normal. ¿Cuál de las siguientes
es la siguiente prueba más apropiada para evaluar la anemia de este
paciente?
a. Aspiración y biopsia de médula ósea.
b. Prueba de antiglobulina directa
c. Recuento de reticulocitos y frotis de sangre periférica.
d. Niveles séricos de hierro y ferritina.
e. Niveles de vitamina B12 y folato.
Abordaje diagnóstico de la anemia en adultos (uptodate)
• Mujer de 24 años es evaluada por dolor en pantorrilla derecha desde
ayer. No informa dificultad para respirar, tos ni dolor en el pecho. Su
historial médico no es destacable y no toma medicamentos. Al examen
físico los signos vitales son normales. Se palpa un cordón venoso
doloroso en la pantorrilla derecha. No se observa asimetría en el
diámetro de la pantorrilla. La ecografía dúplex de la pierna derecha
muestra una trombosis aguda de la vena safena menor derecha de 7 cm
de longitud. ¿Cuál de los siguientes es el manejo más adecuado?
a. Anticoagulación durante 3 meses
b. Anticoagulación durante 6 semanas
c. Ligadura de venas
d. Safenectomía
e. Observación con vigilancia ecográfica
• Mujer de 24 años es evaluada por dolor en pantorrilla derecha desde
ayer. No informa dificultad para respirar, tos ni dolor en el pecho. Su
historial médico no es destacable y no toma medicamentos. Al examen
físico los signos vitales son normales. Se palpa un cordón venoso
doloroso en la pantorrilla derecha. No se observa asimetría en el
diámetro de la pantorrilla. La ecografía dúplex de la pierna derecha
muestra una trombosis aguda de la vena safena menor derecha de 7 cm
de longitud. ¿Cuál de los siguientes es el manejo más adecuado?
a. Anticoagulación durante 3 meses
b. Anticoagulación durante 6 semanas
c. Ligadura de venas
d. Safenectomía
e. Observación con vigilancia ecográfica
Trombosis venosa superficial y flebitis de las venas de las extremidades
inferiores (uptodate)
• Un hombre de 41 años es evaluado en el servicio de urgencias por fatiga
creciente y disnea con el esfuerzo. Recientemente le diagnosticaron cistitis
aguda no complicada y comenzó un tratamiento con nitrofurantoína hace 3
días. En el examen físico, la temperatura es normal, la presión arterial es de
106/72 mm Hg, la frecuencia del pulso es de 104/min y la frecuencia
respiratoria es de 16/min. La saturación de oxígeno es del 98% respirando aire
ambiente. Tiene ictericia conjuntival. Aparte de la taquicardia, el examen físico
es normal. Estudios de laboratorio: Haptoglobina indetectable, hemoglobina 8,8
g/dL, recuento de leucocitos 10.000/uL, recuento de plaquetas 200.000/uL,
recuento de reticulocitos 18% de los eritrocitos, lactato deshidrogenasa 458
U/L. La prueba directa de antiglobulina es negativa para IgG y C3. El frotis de
sangre periférica muestra microesferocitos, esquistocitos y células de ampolla.
Se suspende la nitrofurantoína. ¿Cuál de las siguientes es la gestión adicional
más adecuada?
a. Transfusión de eritrocitos
b. Intercambio de plasma
c. Prednisona
d. Rituximab
e. Monitoreo de síntomas
• Un hombre de 41 años es evaluado en el servicio de urgencias por fatiga
creciente y disnea con el esfuerzo. Recientemente le diagnosticaron cistitis
aguda no complicada y comenzó un tratamiento con nitrofurantoína hace 3
días. En el examen físico, la temperatura es normal, la presión arterial es de
106/72 mm Hg, la frecuencia del pulso es de 104/min y la frecuencia
respiratoria es de 16/min. La saturación de oxígeno es del 98% respirando aire
ambiente. Tiene ictericia conjuntival. Aparte de la taquicardia, el examen físico
es normal. Estudios de laboratorio: Haptoglobina indetectable, hemoglobina 8,8
g/dL, recuento de leucocitos 10.000/uL, recuento de plaquetas 200.000/uL,
recuento de reticulocitos 18% de los eritrocitos, lactato deshidrogenasa 458
U/L. La prueba directa de antiglobulina es negativa para IgG y C3. El frotis de
sangre periférica muestra microesferocitos, esquistocitos y células de ampolla.
Se suspende la nitrofurantoína. ¿Cuál de las siguientes es la gestión adicional
más adecuada?
a. Transfusión de eritrocitos
b. Intercambio de plasma
c. Prednisona
d. Rituximab
e. Monitoreo de síntomas
Diagnóstico y tratamiento de la deficiencia de glucosa-6-fosfato
deshidrogenasa (G6PD) (uptodate)
• Un hombre de 55 años es hospitalizado por confusión y dolor abdominal. El historial
médico es notable por enfermedad hepática relacionada con el alcohol. Toma lactulosa
y espironolactona. En la exploración física el paciente está somnoliento pero
despertable. La temperatura es de 37,7 °C. La presión arterial es de 90/50 mm Hg, la
frecuencia del pulso es de 110/min y la frecuencia respiratoria es de 17/min. Se
observan equimosis en las extremidades superiores e inferiores. Hay ascitis y el
abdomen está difusamente doloroso a la palpación. Estudios de laboratorio: Tiempo de
tromboplastina parcial activada 35 s, dímero D 0,7 mg/L, hemoglobina 9,4 g/dL,
recuento de leucocitos 12.000/uL, recuento de plaquetas 68.000/uL, tiempo de
protrombina 17 s, fibrinógeno 80 mg/dl. La ecografía abdominal demuestra una
apariencia nodular del hígado, esplenomegalia moderada y gran cantidad de ascitis. La
paracentesis confirma el diagnóstico de peritonitis bacteriana espontánea y se inicia el
tratamiento. ¿Cuál de las siguientes pruebas será más útil en la evaluación de la
coagulopatía del paciente?
a. Nivel del factor II
b. Nivel de factor VIII
c. Nivel del factor X
d. Tiempo de trombina
e. Observación
• Un hombre de 55 años es hospitalizado por confusión y dolor abdominal. El historial
médico es notable por enfermedad hepática relacionada con el alcohol. Toma lactulosa
y espironolactona. En la exploración física el paciente está somnoliento pero
despertable. La temperatura es de 37,7 °C. La presión arterial es de 90/50 mm Hg, la
frecuencia del pulso es de 110/min y la frecuencia respiratoria es de 17/min. Se
observan equimosis en las extremidades superiores e inferiores. Hay ascitis y el
abdomen está difusamente doloroso a la palpación. Estudios de laboratorio: Tiempo de
tromboplastina parcial activada 35 s, dímero D 0,7 mg/L, hemoglobina 9,4 g/dL,
recuento de leucocitos 12.000/uL, recuento de plaquetas 68.000/uL, tiempo de
protrombina 17 s, fibrinógeno 80 mg/dl. La ecografía abdominal demuestra una
apariencia nodular del hígado, esplenomegalia moderada y gran cantidad de ascitis. La
paracentesis confirma el diagnóstico de peritonitis bacteriana espontánea y se inicia el
tratamiento. ¿Cuál de las siguientes pruebas será más útil en la evaluación de la
coagulopatía del paciente?
a. Nivel del factor II
b. Nivel de factor VIII
c. Nivel del factor X
d. Tiempo de trombina
e. Observación
Anomalías hemostáticas en pacientes con enfermedad hepática (uptodate)
ucontinental.edu.pe

También podría gustarte